A nervous 42-year-old woman presents herself to your antenatal clinic very worried that she has missed the right time to have her combined test for Down’s syndrome screening. She is now 17 weeks pregnant and is very concerned about her age. You counsel her about the appropriate alternative, the quadruple test and arrange to have this done. What assays make up the quadruple test?
A 33-year-old nulliparous woman is 29 weeks pregnant. She was referred to the rapid access breast clinic for investigation of a solitary breast lump. Sadly, a biopsy of this lump revealed a carcinoma. After much counselling from the oncologists and her obstetricians a decision is reached on her further treatment. What option below may be available to her?
A 38-year-old woman with type 2 diabetes attends the maternal medicine clinic. She has a body mass index (BMI) of 48 and is currently controlling her sugars with insulin. You have a long discussion about her weight. What should not be routinely offered to this woman?
A nulliparous woman is seen at the antenatal clinic 27 weeks into her first pregnancy. Routine screening with a 75 g oral glucose tolerance test for gestational diabetes mellitus (GDM) is performed. Which of the following would confirm a diagnosis of GDM?
A 29-year-old woman is seen at her booking visit and has blood taken for screening. Which of these is the most appropriate set of booking tests?
A 34-year-old woman attends antenatal clinic for a routine ultrasound scan. Abnormalities of placentation are detected and a magnetic resonance imaging (MRI) scan is organized by the fetal medicine consultant. The MRI report shows: ‘The placenta is in the lower anterior uterine wall with evidence of invasion to the posterior wall of the bladder’. What is the most likely diagnosis?
A 30-year-old nulliparous woman is 29 weeks pregnant. She presented to hospital with a history of a minor, unprovoked painless vaginal bleed of about a teaspoonful. Her anomaly scan at 20 weeks showed a low-lying placenta. Her fetus is moving well and continuous cardiotocography (CTG) is reassuring. What is the most appropriate management?
A 28-year-old pregnant woman attends accident and emergency with a history of clear vaginal loss. She is 18 weeks pregnant and so far has had no problems. Her past medical history includes a large cone biopsy of the cervix and she is allergic to penicillin. She is worried because the fluid continues to come and there is now some blood. On examination it is apparent that her membranes have ruptured. What is the most appropriate initial management?
A 37-year-old woman in her fourth ongoing pregnancy presents to the labour ward at 34 weeks’ gestation complaining of a sharp pain in her chest, worse on inspiration. An arterial blood gas shows: pH 7.51, PO2 8.0 kPa, PCO2 4.61 kPa, base excess 0.9. What is the most appropriate investigation?
A 32-year-old woman in her second pregnancy presents at 36 weeks gestation with a history of a passing gush of blood stained fluid from the vagina an hour ago, followed by a constant trickle since. The admitting obstetrician reviews her history and weekly antenatal ultrasound scans have shown a placenta praevia. What is the most appropriate management? She has a firm, posterior cervix and has not been experiencing any contractions.
Maternal physiology changes throughout pregnancy to cope with the additional demands of carrying a fetus. Which of the following changes best represents a normal pregnancy?
A 30-year-old woman attends the antenatal clinic asking to be sterilized at the time of her elective caesarean. She is 34 weeks into her second pregnancy having had her first child 2 years ago via an emergency caesarean section. She is not sure that she wants any more children. Further more, she does not wish to try for a vaginal birth. She has tried the contraceptive pill in the past but does not like the side effects. You talk to her about other options, including the sterilization she is requesting. What is the best management option for this woman?
A 41-year-old multipara attends the antenatal clinic at 36 weeks gestation complaining of lower abdominal cramps and fatigue when mobilizing. Clinical examination is unremarkable save for a grade I pansystolic murmur, loudest over the fourth intercostal space in the midaxillary line. What is the most appropriate management?
A 32-year-old HIV positive woman who booked for antenatal care at 28 weeks gestation arrives on the delivery suite at 37 weeks with painful regular contractions and a cervix dilated to 4 cm. Ultrasonography confirms a breech singleton pregnancy with a reactive fetal heart rate. What is the most appropriate management option?
A 41-year-old multiparous woman attends accident and emergency at 32 weeks gestation complaining of sudden onset shortness of breath. A CTPA demonstrates a large saddle embolus. What is the most appropriate treatment regimen?
A 21-year-old woman attends the labour ward with per vaginal bleeding of 100 mL. She is 32 weeks pregnant and has had one normal delivery in the past. An important history to note is that of an antepartum haemorrhage in her last pregnancy and she smokes 10 cigarettes a day. Her 20-week anomaly ultrasound revealed a posterior fundal placenta. She admits she and her partner had intercourse last night and is concerned by terrible abdominal pains. What is the most likely diagnosis?
At a booking visit a first time mother is told that she is rhesus negative. Which of these answers is the most appropriate advice for the mother?
A 42-year-old para 4 with a dichorionic–diamniotic (DCDA) twin pregnancy at 31 weeks gestation presents to hospital with a painful per vaginam bleed of 400 mL. The bleeding seems to be slowing. She is cardiovascularly stable, although having abdominal pains every 10 minutes. There is still a small active bleed on speculum and the cervix appears closed. Both fetuses have reactive CTGs. She has had no problems antenatally and her 28-week ultrasound revealed both placentas to be well away from the cervix. What is your preferred management plan?
You are the FY1 covering the antenatal ward. A 27-year-old nulliparous woman who is 36 weeks and 5 days pregnant has been admitted to your ward with suspected pre-eclampsia. The emergency buzzer goes off in her room. You are the first to attend and find your patient flat on the bed having a generalized seizure – what do you do?
A 38-year-old woman in her first pregnancy is 36 weeks pregnant. She presents to the labour ward feeling dizzy with a mild headache and flashing lights. Her past medical history includes systemic lupus erythematosus (SLE), renal stones and malaria. Her blood pressure is 158/99 mmHg with 2+ protein in her urine. Her platelets are 55 × 10^9/L, Hb 10.1 g/dL, bilirubin 62 μmol/L, ALT 359 IU/L, urea 2.3 mmol/L and creatinine 64 μmol/L. What is the most likely diagnosis?
A 19-year-old woman in her first pregnancy presents to the GUM clinic with an outbreak of primary herpes simplex infection on her labia. She is 33 weeks pregnant. What is the best advice regarding her herpes?
A 33-year-old woman presents to hospital with a 2-day history of itching on the soles of her feet and the palms of her hands. Her pregnancy has been straightforward and she has good fetal movements. Liver function tests reveal an alanine transaminase (ALT) of 64 IU/L and bile acids of 30 μmol/L. You suspect that she might have developed obstetric cholestasis. Which of the following bits of advice is true?
A 24-year-old woman who is 32 weeks pregnant presents to the labour ward with a terrible headache that has not improved despite analgesia. It started 2 days ago and came on suddenly. She has stayed in bed as it hurts to be in sunlight and she vomited twice this morning. Her past medical history includes a macroprolactinoma (which has been removed) and occasional migraines. She is haemodynamically stable with no focal neurology or papilloedema. You arrange for her to have a CT of her head as an emergency, which adds no further information to aid your diagnosis. There are red cells on lumbar puncture but no organisms are isolated. What is the most likely diagnosis?
A 19-year-old woman in her first pregnancy is admitted to the labour ward with a 4-hour history of lower abdominal pain – she is 22 weeks pregnant. She has not had any vaginal bleeding but describes a possible history of rupture of her membranes. Her past medical history includes an appendectomy and a large cone biopsy of her cervix. On examination she has palpable lower abdominal tenderness, her cervix is 2 cm dilated, she has an offensive vaginal discharge and her temperature is 38.9ºC. Her white cell count is 19.0 × 10^9/L and her C-reactive protein is 188 mg/L. There are no signs of cardiovascular compromise. How would you manage this woman?
A 24-year-old multiparous woman is 23 weeks pregnant. She has not had chicken pox before. She goes to a collect her 3-year-old son from a birthday party and comes into contact with a child with an infective chicken pox infection. She is naturally very anxious. What is the best course of management?
A 32-year-old woman in her third pregnancy is 37 weeks pregnant and has an extended breech baby on ultrasound. After discussion in the antenatal clinic, which of the following is not an absolute contraindication to an external cephalic version (ECV)?
A 24-year-old type 1 diabetic woman has just had her first baby delivered by caesarean section at 35 weeks due to fetal macrosomia and poor blood sugar control. The operation is straightforward with no complications. She has an insulin sliding scale running when you review her on the ward 12 hours postoperatively. She has begun to eat and drink. How would you manage her insulin requirements?
A 19-year-old woman is referred to your pre-conception clinic. She has SLE and wants to fall pregnant. She is currently not on any treatment and has no symptoms. As part of your general counselling you should talk about the risks associated with pregnancy. Which of the following is not a particular risk to a woman with SLE?
A 44-year-old women who is 18 weeks pregnant presents to your clinic with a 2-day history of a viral illness. She is extremely anxious and is in floods of tears. She recently had some soft cheese in a restaurant and after an internet search she is convinced she has a particular infection. What infection is she concerned about?
A 26-year-old woman is 37 weeks pregnant and consults you about a rash that started on her abdomen and has now spread all over her body. Interestingly her umbilicus is spared. The rash is very itchy and nothing is helping. The rash is her first problem in this pregnancy. Of interest, her mother has pemphigoid and her sister has psoriasis. What is the most likely cause of her rash?
Which of the following drugs is not absolutely contraindicated in pregnancy?
A 42-year-old woman is in her first pregnancy. She conceived with in vitro fertilization (IVF) and has had a straightforward pregnancy so far. At 25 weeks’ gestation she is seen in clinic with a blood pressure of 142/94 mmHg and protein + in her urine. A protein creatinine ratio (PCR) comes back as 19. She says that her blood pressure is often up at the doctor’s. With the information you have to hand what is the most likely diagnosis?
A 24-year-old woman attends the antenatal clinic. She has had a glucose tolerance test which is abnormal. A diagnosis of gestational diabetes is made. The primary purpose of this appointment is to explain to her what gestational diabetes means to her and her baby. You explain to her that sugar control is important and there are specific glucose ranges that she should try to adhere to. Which of the following would be correct advice for this woman?
A 24-year-old woman in her first pregnancy has a significantly raised glucose tolerance test at 28 weeks gestation: 4.6 fasting 12.1 at one hour 9.1 at 2 hours (μmol/L). She is given the diagnosis of GDM. You are asked to counsel her about the effects of gestational diabetes on pregnancy. Which of the following is not an additional effect of having GDM?
A 24-year-old woman who is HIV positive is in her first pregnancy. She is 39 weeks pregnant and is seen by you in the antenatal clinic. She has just transferred to your care, with no other previous antenatal care. She reports that her pregnancy has been uncomplicated. Her CD4 count is 180/mm3 and her viral load is 5500 copies/mL. She has come to find out what advice you have for her delivery.
A 24-year-old woman attends accident and emergency 4 weeks after having a positive urinary pregnancy test. She has had 3 days of painless vaginal bleeding and is passing clots. Over the past 2 days the bleeding has settled. An ultrasound scan shows an empty uterus. What is the correct diagnosis?
A 51-year-old woman in her 12th week of an assisted-conception triplet pregnancy presents to accident and emergency with severe nausea and vomiting. She has mild lower abdominal and back pains. Urine dipstick shows blood –ve, protein –ve, ketones ++++, glucose +. What is the most appropriate management plan?
A 19-year-old woman is referred to your early pregnancy unit as she is having some vaginal bleeding. This is her first pregnancy, she has regular menses and the date of her last menstrual period suggests she is 8 weeks gestation today. She is well apart from her bleeding and is naturally concerned. A transvaginal ultrasound reveals an intrauterine gestational sac of 18 mm with a yolk sac. What is the most likely explanation of these findings?
A 31-year-old woman is seen in the termination of pregnancy (TOP) clinic requesting a termination. She is 5 weeks pregnant in her first pregnancy. She is otherwise well but does have some lower abdominal pain on the right hand side. On examination her abdomen is soft and non-tender. An ultrasound reveals a small sac in the uterus which might be a pseudosac. What would be your next management step?
A 28-year-old woman with a history of pelvic inflammatory disease is 6 weeks into her third pregnancy. She previously had two terminations. She presents with lower abdominal pain and per vaginam bleeding. Her beta hCG is 1650 mIU/mL, progesterone 11 nmol/l. An ultrasound reveals a small mass in her left fallopian tube with no intrauterine pregnancy seen. There is no free fluid in the Pouch of Douglas. She is diagnosed with an ectopic pregnancy and is clinically stable but scared of surgery. How would you manage this case?
A 24-year-old woman attends her GP complaining of deep dyspareunia and post-coital bleeding. She has crampy lower abdominal pain. Of note, she has been treated in the past for gonorrhoea on more than one occasion. On speculum examination there is no visible discharge, but the cervix bleeds easily on contact. What is the most appropriate management?
A 16-year-old girl attends accident and emergency complaining of mild vaginal spotting. Her serum beta hCG is 4016 mIU/mL. She is complaining of severe left iliac fossa pain and stabbing sensations in her shoulder tip. What is the most appropriate definitive investigation?
An 18-year-old woman presents to accident and emergency having fainted at work. She is complaining of pain in the lower abdomen. A serum beta hCG performed in the emergency department is 3020 mIU/mL. The on-call gynaecologist performs transvaginal ultrasonography in the resuscitation area which shows free fluid in the Pouch of Douglas and no visible intrauterine pregnancy. Her pulse is 120 bpm and blood pressure 90/45 mmHg. What is the most likely diagnosis?
A 50-year-old woman comes to your clinic with a 2-year history of no periods. Her GP has confirmed that her luteinizing hormone and follicle-stimulating hormone levels are menopausal. Her night sweats and hot flushes are unbearable and are preventing her from going to work. She would like to start hormone replacement therapy (HRT) but is very worried about the side effects. Which of the following is incorrect?
A 24-year-old woman who is 9 weeks pregnant is brought to accident and emergency by ambulance with left iliac fossa pain and a small vaginal bleed. An abdominal ultrasound scan performed at the bedside demonstrates a cornual pregnancy and free fluid in the pelvis. Her observations are: pulse 119 bpm, blood pressure 74/40 mmHg, respiratory rate 24/minute. What is the most appropriate definitive management?
A 26-year-old woman presents to accident and emergency with left-sided lower abdominal pain and a single episode of vaginal spotting the day before. A urinary beta hCG is positive, and her last period was 6 weeks ago. A transvaginal ultrasound shows two gestational sacs. What is the most likely diagnosis?
A 32-year-old woman with paranoid schizophrenia is admitted for antenatal assessment at 36 weeks’ gestation with twins. Her pregnancy is complicated by intrauterine growth restriction and impaired placental flow. She has had no psychotic symptoms in this pregnancy. Her obstetricians recommend an early caesarean section and argue it is in the best interests of the mother and her babies and to prevent further fetal insult. She has repeatedly said that despite the significant risks, which she understands, she refuses caesarean delivery. What is the most appropriate action?
Which of the following would be incorrect advice to give a woman requesting a caesarean section for non-medical indications?
A 24-year-old Jehovah’s Witness is brought to accident and emergency with a Glasgow coma scale (GCS) score of 3, BP 90/30 mmHg and pulse 110 bpm. Her husband reports that her last menstrual period was 8 weeks ago and she complained this morning of lower abdominal pain and vaginal spotting. Ultrasonography suggests a ruptured ectopic pregnancy. As part of the resuscitative measures employed before emergency laparotomy, a transfusion of group O-negative blood is prepared. Her husband interrupts and says that as a Jehovah’s Witness she would absolutely refuse all blood products even at risk of death, and has previously signed an advance directive stating this. What is the most appropriate option?
An unbooked 26-week pregnant woman sees you at the hospital to request a termination of pregnancy. She says that if she leaves here today without a termination she will try and do it herself by stabbing her abdomen. Your consultant arranges an urgent psychiatric review which finds no grounds under which to detain this woman in regards to her mental health. Under these circumstances, if a termination was performed, which part of the Abortion Act would it fall under?
A 24-year-old woman in her first pregnancy presents to the labour ward in labour. She and her partner express an overwhelming desire to avoid a caesarean section. Her labour does not progress and after 9 hours her cervix is still only 3 cm dilated. Unfortunately, the fetal heart slows to 60 beats and does not recover after 5 minutes. Your senior registrar explains the situation to the woman and recommends an immediate caesarean section. She refuses and her partner tells you to stop harassing them. You explain that their unborn child will die if this continues. What options do you have?
A 16-year-old Muslim woman attends accident and emergency department with her father. She complains of a 1-day history of left iliac fossa pain and mild vaginal spotting. A urinary beta hCG test is positive. As part of your assessment the patient consents to a vaginal examination. She insists you do not tell her father that she is pregnant, and you consider her to be competent in her judgement. Her father becomes angry and says you must not perform a vaginal examination. How should you proceed?
A 32-year-old woman is rushed to accident and emergency as the viction of a high speed vehicle collision. She is 35 weeks pregnant and unconscious. There is evidence of blunt abdominal trauma and she is showing signs of grade 3 hypovolaemic shock. The consultant obstetrician on call immediately attends the resus call and recommends immediate perimortem caesarean delivery in a resuscitative effort to improve the management of her shock. Her husband has been brought into resus by the police, and insists that she would refuse caesarean section under any circumstances. What is the most appropriate management?
A 59-year-old woman has been admitted for a hysterectomy for endometrial cancer. She has not yet given her consent and the rest of the team is in theatre. You have performed a hysterectomy before so feel confident in taking her through what will happen and the risks involved. The General Medical Council (GMC) says that you should tailor your discussion to all of the options except which of the following?
A quality improvement process that seeks to improve patient care and outcomes through systematic review of care against explicit criteria and the implementation of change. This is an accepted definition of what?
A 15-year-old girl attends the gynaecology clinic with her boyfriend, also 15, requesting the morning after pill 4 months after being circumcised during a family trip to Somalia. She understands your advice and the implications of her decisions to engage in sexual activity, is using condoms regularly and refuses to inform her parents. What is the most appropriate management?
A 59-year-old woman attends the gynaecology clinic complaining of worsening pain during penetrative sexual intercourse. She went through the menopause 9 years before, with very few problems, and did not require hormone replacement therapy (HRT). She has been with the same partner for 4 years since the death of her husband with whom she had four children. What is the most likely diagnosis?
A 19-year-old woman is referred to accident and emergency with a fluctuant lower right abdominal pain which started over the course of the morning, associated with vomiting. There is rebound tenderness on examination. She is afebrile. Serum beta human chorionic genadotrophin (hCG) is negative. An ultrasound shows free fluid in the peritoneal cavity but no other pathology to account for the pain. White cells are 14 × 10⁹/L and the C-reative protein (CRP) is 184 mg/L. What is the most likely diagnosis?
A 39-year-old woman is seen in the gynaecology clinic having been diagnosed with polycystic ovarian syndrome (PCOS). She has lots of questions in particular about the associated long-term risks. Which of the following is not a risk of PCOS?
A 54-year-old menopausal woman comes to your clinic desperate for hormone replacement therapy (HRT) as her vasomotor symptoms are very troubling. Her next door neighbour recently developed a deep vein thrombosis while on HRT. She is concerned about the risks of venous thromboembolism (VTE) and wants your advice. Which of the following would you not advise?
A 34-year-old woman with long-standing menorrhagia attends accident and emergency having fainted at home. She is on the third day of her period, which has been unusually heavy this month. She insists she cannot be pregnant as she has not had sexual intercourse for a year. She is haemodynamically stable. A point-of- care test venous full blood count in the emergency department shows: Hb 5.2 g/dL WCC 8.9 × 10⁹/L Hct 0.41% L MCV 80 fL What should the initial management be?
A 66-year-old post-menopausal woman is referred to you urgently by her general practioner (GP). She had been complaining of some lower abdominal pain. An ultrasound arranged by the GP shows a 4 cm simple left ovarian cyst. A CA 125 comes back as 29 U/ml (normal 0-35 U/ml). What is the most appropriate management?
A 79-year-old woman attends your clinic with some vaginal bleeding. Her last period was 16 years ago. She has had two children both via caesarean section, has a normal smear history and is currently sexually active. On examination the vagina appears mildly atrophic with some raw areas near the cervix. What is the most important next step in her management?
At laparoscopy a 21-year-old woman is found to have severe endometriosis. There are multiple adhesions and both ovaries are adherent to the pelvic side wall. The sigmoid colon is adherent to a large rectovaginal nodule. The nodule is excised and the bowel and ovaries freed. Which of the following medications would be appropriate to help treat her endometriosis?
A 54-year-old woman comes to your clinic complaing of hot flushes and night sweats that are unbearable. Her last mentrual period was 14 months ago. She has had a levonorgestrel releasing intrauterine system (Mirena) in situ for 2 years as treatment for extremely heavy periods. What treatment would you consider for her symptoms?
A 19-year-old biochemistry student is seen in your clinic worried about her hormone levels. She has been told by her GP that her progesterone is low. You enter into a long discussion about the effects of progesterone on the body. Progesterone:
A 41-year-old mother of two presents to the GP with long-standing heavy menstrual bleeding which has become worse over the past year. She is otherwise well and has no significant medical history. She requests treatment to alleviate the impact of her heavy bleeding on her social life. Pelvic examination reveals a normal sized uterus. What is the most appropriate first line treatment?
A 42-year-old woman is seen in the gynaecology clinic. She has been suffering from severe premenstrual symptoms all her life. They have now significantly affected her relationship and her husband is filing for divorce. She comes to your clinic in tears regarding the future of her children. She demands a hysterectomy and bilateral salpingoophrectomy. After taking her history you talk about other less radical treatments. Which management option is inappropriate?
A 22-year-old woman is seen in accident and emergency with lower abdominal pain and some vaginal discharge. She has had PID once in the past and was treated for it. She is otherwise well. Her temperature is 36.9°c, pulse 90, blood pressure 105/66 mmHg. She is passing good volumes of urine. On clinical examination she has diffuse lower abdominal tenderness. There are no signs of peritonism on examing her abdomen. On vaginal examination she has adnexal tenderness and an offensive discharge. Her CRP is 28 mg/L and her white blood count is 12.2 × 10⁹/L. Her pregnancy test is negative. She is reviewed by your senior and is diagnosed with PID. What would be an appropriate antibiotic regime?
A 24-year-old woman is in her first pregnancy. She has no significant medical history. She is 40 weeks and 2 days pregnant and has been contracting for 4 days. She is not coping with the pain. She has been given intramuscular pethidine. On examination she is found to be 4 cm dilated (fetus in the occipito-posterior position) having been the same 4 hours previously. What analgesia would you recommend?
A 36-year-old woman is 41 weeks pregnant and is established in spontaneous labour. She is contracting three times every 10 minutes and has ruptured her membranes. She is draining significant meconium stained liquor. Her cervix is 7 cm dilated. Her midwife has started continuous electronic fetal monitoring using a cardiotocograph (CTG). The baseline rate has been 155, with variability of 2 beats per minute, for the past 60 minutes. There are no accelerations and no decelerations. What is the most appropriate management?
A 19-year-old woman is giving birth to her first baby. She has been pushing for an hour and the fetal head has been on the perineum for 6 minutes. There seems to be a restriction due to resistance of her tissues. Her midwife carries out a right mediolateral episiotomy. Which of the following structures should not be cut with the episiotomy?
A 25-year-old woman in her first pregnancy has a pathological CTG. Her cervix is 5 cm dilatated. Which of the following might increase the risk to the fetus if the doctor performed a fetal blood sample?
A multiparous woman is admitted to the labour ward with regular painful contractions. On examination she is 9 cm dilated with intact membranes and is coping well with labour pains. Forty minutes later her membranes rupture while she is being examined and you see the umbilical cord hanging from her vagina. You inform the woman what has happened. She is now fully dilated, the fetal position is Direct occipitoanterior, and the presenting part is below the ischial spines. What do you do next?
A 34-year-old para 0 has been admitted for a post-dates induction of labour at 42 weeks. She has received 4 mg PGE₂ (prostaglandin) vaginally. After 72 hours her cervix is 5 cm dilated. Four hours later she is still 5 cm dilated. On abdominal examination the fetus appears to be a normal size. The fetal head position is left occipito-transverse, and the station is −1. There is no moulding but a mild caput. She is contracting two times in every 10 minutes and has an epidural in situ. You are asked to review and make a management plan. What would be the most appropriate plan?
A mother comes to labour ward who is low risk, in labour at term. The unit is short staffed and there are not enough midwives to provide intermittent auscultation of the fetal heart. You decide to start continuous electronic monitoring (CTG). She is an epidemiologist and asks you about the CTG and how it will help her labour and prevent her baby suffering harm. Which of the following would you tell her? Continuous monitoring has a:
A 29-year-old woman comes to the labour ward complaining that her baby has not been moving for 72 hours. She is 36 weeks pregnant. Otherwise her pregnancy has been complicated with gestational diabetes for which she is taking insulin. On examination you fail to pick up the fetal heart. You confirm the diagnosis of an intrauterine death. The scan shows no liquor and the baby is transverse. After a long discussion you explain that she unfortunately needs to deliver her baby. What is the best way for her to deliver her baby?
A 24-year-old woman with gestational diabetes has been progressing normally through an uncomplicated labour. The midwife delivers the head but it retracts and does not descend any further. What should the midwife do next?
A 29-year-old multiparous woman is in established labour contracting strongly. She is 4 cm dilated and had been having regular painful contractions for 6 hours before they stopped abruptly, heralded by a sudden onset of severe, continuous lower abdominal pain. The fetal heart trace is difficult to identify, and the tocometer does not register a signal. What is the most appropriate management?
A 23-year-old woman is in her first labour. Her cervix is 6 cm dilated and she is in distress. She is asking for an epidural. Before you call the anaesthetist you check her history. Which of the following would be an absolute contraindication to an epidural?
A 38-year-old nulliparous woman has had an uncomplicated pregnancy. She has laboured very quickly and is 10 cm dilated. The fetal heart falls to 60 for 4 minutes. She is pushing effectively and the head is 1 cm below the ischial spines. You prepare for forceps delivery in the room. She has had no analgesia so you quickly insert a pudendal nerve block and deliver the baby 4 minutes later in good condition. Which of the following is not a branch of the pudendal nerve?
The obstetric team is alerted to a blue-light trauma call expected in accident and emergency. A 28-year-old woman who is 37 weeks pregnant has been involved in a high-speed road traffic collision. On arrival, where the obstetric team is on standby, her Glasgow Coma Scale score is 5 and she has a tachycardic hypotension. What is the most appropriate management sequence?
A 24-year-old woman is seen after her normal vaginal birth. The midwife who delivered the baby is concerned that there is a third degree tear. Having examined the woman the obstetrician confirms a third degree tear. The woman is taken to theatre to repair the external anal sphincter. Which of the following is not a risk factor for third degree tear?
A 34-year-old woman is brought straight to intensive care from the obstetric theatre after an emergency caesarean section for fetal distress. The attending obstetrician remarks that she is showing haematological signs of disseminated intravascular coagulation. Which blood profile is she most likely to have?
A 31-year-old undergoes a planned caesarean section for a breech presentation. After delivery of her healthy baby there is difficulty in delivering the placenta, as it is adhered to the uterus. She has lost 5 L of blood as a result of the placenta accreta. The placenta has been removed but she is still bleeding and is cardiovascularly unstable despite blood product replacement. What would be the most management to definitively arrest haemorrhage?
A 39-year-old woman is 6 days post-partum and has come back to hospital with shortness of breath. She is struggling to breath at rest, has a respiratory rate of 28, pulse 115, BP 105/60 mmHg, temperature 37.4° C. On examination she has an audible wheeze and cough. Investigations reveal a PO₂ of 9.5 kPa on arterial blood gas and a PCO₂ 3.7 kPa, pH 7.36, base excess -3.4. A chest x-ray shows some upper lobe diversion and bilateral diffuse shadowing with an enlarged heart. Her haemoglobin is 8.9 g/dL, white blood count 11.1 × 10⁹/L and C-reactive protein 21 mg/L. What is the most likely cause of her symptoms?
A 17-year-old girl is seen in accident and emergency 14 days after an emergency caesarean delivery of a healthy infant, her first. Her neighbours became concerned and called the police. She had been seen prostrate in the garden chanting verses from the Bible and shouted at them accusing them of being spies when they asked if she was ok. They say her problem has worsened over the past fortnight. What is the most likely diagnosis?
At birth, which of the following does not occur in the fetal circulation?
A woman on the labour ward has just had a normal birth. At birth there was a lot of meconium present. The newborn did not respond initially but did after subsequent resuscitation. The midwife records the Apgar score as 5. Which of the following best describes the categories an Apgar score is created from?
An 18-year-old woman has been successfully delivered of a healthy female infant by elective caesarean section for maternal request. Estimated blood loss was 1120 mL. Forty minutes after return to the recovery area, she has a brisk vaginal bleed of around a litre. Her pulse rate is 120 bpm and blood pressure is 95/55 mmHg. What should the immediate management process be?
A 34-year-old woman develops a significant post-partum haemorrhage and hypotensive shock following vaginal delivery of a healthy infant at term. The labour was uncomplicated. She recovers well with volume replacement and oxytocin and returns to the post-natal ward. She is unable to breast feed on the ward and 2 months later has neither started breastfeeding nor resumed her periods and is increasingly fatigued. What is the most likely diagnosis?
A 30-year-old French woman delivers a live female infant by spontaneous vaginal delivery at term. In the eleventh week of pregnancy she developed a flu-like illness which resolved spontaneously a week later. Her newborn child has severe hydrocephalus and chorioretinitis. Four days after birth, she develops severe convulsions and efforts to revive her are unsuccessful. Which pathogen is most likely to be responsible?
A 32-year-old woman has a routine cervical smear at her GP practice. The result returns as severe dyskaryosis. Following colposcopy and cervical biopsy, formal histological examination reveals cervical intraepithelial neoplasia 3 (CIN 3). Which of the following pathogens is the most likely to have caused this disease?
A 15-year-old girl attends the paediatric gynaecology clinic with primary amenorrhoea and features of secondary breast development. She has intermittent abdominal bloating and is extremely worried that she is ‘not like other girls’. On speculum examination of the vagina, which is normal externally, a bulging red disc is seen 3 cm proximal to the introitus. What is the most likely diagnosis?
A 19-year-old woman undergoes surgical evacuation of the retained products of conception (ERPC). Histological examination of the sample shows genetically abnormal placenta with a mixture of large and small villi with scalloped outlines, trophoblastic hyperplasia. What is the most likely diagnosis?
An 89-year-old woman attends the gynaecology clinic with a long history of a dragging sensation in the vagina. Apart from severe aortic stenosis, she has no significant medical history. She leaks fluid when she sneezes or coughs. On examination with a Sims’ speculum in the left lateral position, a grade 1 uterine prolapse is seen, with an additional cystocoele. What is the most appropriate management?
A 46-year-old woman presents to your clinic with a 6-year history of Incontinence. She has had four children by vaginal deliveries, has a body mass index (BMI) of 35 kg/m² and suffers from hayfever. Initial examination reveals a very small cystocele. A mid-stream urine culture is negative and urodynamic studies show a weakened urethral sphincter. What is the most appropriate first line management?
A 16-year-old girl attends the gynaecology clinic complaining of vaginal itching and lumpy labia. On examination the area is covered with vulval warts. Which is the causative pathogen for vulval warts?
A 25-year-old woman attends accident and emergency with an exquisitely sore, large swelling of her vagina which she noticed only a couple of days before. It has steadily got much bigger. On examination there is a soft fluctuant mass on the right labia minora which is very tender. What is the most appropriate management?
An 18-year-old woman attends clinic seeking contraceptive advice. She is currently using condoms only and is keen to start taking the combined oral contraceptive pill (COCP). Her sister used to take it but told her there were lots of problems with it. Her aunt has bowel cancer and she has no other past medical history. Appropriate counselling should cover all of the following except:
A 49-year-old woman presents to a private clinic expressing her desire to become pregnant. She has no past medical history. Initial investigations show that she still has ovarian function, is ovulating and is having regular periods. An ultrasound of her pelvis shows no structural abnormality and an hysterosalpingography demonstrates patent fallopian tubes. Analysis of her partner’s semen is normal. Which would not be an appropriate first line management option?
A 42-year-old woman presents to the urogynaecology clinic with a 3-year history of urge incontinence. She has features of an overactive bladder and is desperate to start treatment for her problem as it is affecting her quality of life. She opts for medical treatment. What is the most appropriate first line pharmacological therapeutic?
A 41-year-old woman is about to undergo her first cycle of IVF. As part of the consultation, she is counselled about the maternal and fetal risks involved with IVF-conceived pregnancies. All of the following occur in such pregnancies except:
A 16-year-old presents to the termination of pregnancy service 6 weeks into her second pregnancy requesting surgical termination (STOP). What is not required as part of her work-up for the procedure?
A 35-year-old woman is seen in the assisted conception unit. She has been trying to conceive for 4 years. In this period she has been having regular intercourse. Her periods have been irregular and recently she has had no periods at all. Her BMI is 19.5 kg/m², she has had an appendectomy and is otherwise well. Her biochemistry comes back as follows: luteinizing hormone (LH) 0.5 IU/L, follicle-stimulating hormone (FSH) 1.0 IU/L, prolactin 490 mIU/L, thyroxine (T4) 12, thyroid stimulating hormone (TSH) 4.2 mIU/L, oestradiol 60 pmol/L. What is the most likely cause of her subfertility?
A 19-year-old comes to you for some pre-conception advice. Some members of her family and her partner’s family have a sickle cell anaemia. She reveals that her sister and his sister are both affected. Tests have shown that they are both carriers. What is the chance that if their child was a boy he would have sickle cell anaemia?
An 18-year-old girl is seen in the colposcopy clinic after having had persistent post-coital bleeding. She has been sexually active since the age of 14 and has no past medical history. She is studying for her A-levels and has been doing a lot of reading. She is concerned that she might have cervical cancer. Which of the following is not a risk factor for cervical cancer?
A 49-year-old comes to the urogynaecology clinic with a history of leaking urine for the last year. There are associated stress symptoms and some urge symptoms. Interestingly she says that it seems to come from inside the vagina as well. She had a hysterectomy last year for endometrial cancer and had quite a prolonged recovery. She has a BMI of 30 kg/m², does not smoke and is otherwise fit and well. You are suspicious that she might have a vesico- vaginal fistula secondary to her operation. What is the most appropriate first line investigation?
A 16-year-old girl presents to your surgery with a history of unprotected sexual intercourse (UPSI) 70 hours ago. Her last menstrual period was 8 days ago. Her only past medical history of note is that of epilepsy which is well controlled by carbamazepine. She is worried about becoming pregnant, does not want her mother to find out and is in a hurry to get home before suspicions are raised. Which of the following options are available to her?
A 40-year-old woman comes to your clinic alone wanting an effective form of contraception. She has two children from a previous marriage and has recently started a new relationship. She says that she does not want any further children. She has regular heavy periods, no menopausal symptoms and she is otherwise well with no past medical history. A recent ultrasound showed a normal sized uterus and pipelle biopsy revealed normal secretory endometrial tissue. What is the most appropriate form of contraception?
In a busy gynaecology clinic you are assessing a 22-year-old woman who has not had a period for 18 months. She is not pregnant and previously had regular periods. She has had two surgical terminations of pregnancies (STOP), an underactive thyroid gland and an appendectomy. Clinical examination is unremarkable with a BMI kg/m² of 20. Biochemical investigations reveal a T4 of 17 pmol/L, TSH 4.6 kg/m², prolactin of 570 mU/L, and testosterone of 42 ng/dL. LH and FSH are normal. Vaginal ultrasound shows a normal sized uterus and the left ovary contain four cysts. Which of the answers listed below is the most likely cause?
A 26-year-old woman is otherwise fit and well has been trying to conceive for over 2 years. On questioning she has regular periods and has been having regular intercourse. There are no abnormalities on clinical examination. What would be your first line investigations for her subfertility?
A 42-year-old man undergoes semen analysis as part of the investigation of subfertility with his wife. What result would most likely contribute to their subfertility?
A 46-year-old women in her fifth IVF cycle is admitted to the emergency department 4 days after egg collection. She is complaining of a swollen abdomen and shortness of breath. She is reviewed and a diagnosis of ovarian hyperstimulation syndrome (OHSS) is made. Which of the following is not a clinical feature/complication of OHSS?
A 17-year-old girl comes to clinic with her mother as she has not started having periods yet and they are worried. On examination she is of short stature, with a slightly widened neck and has no secondary sexual characteristics and there is no obvious abnormality of the external genitalia. What is the most likely diagnosis form this limited information?
A 22-year-old woman presents to the GUM clinic with an offensive smelling discharge. She is sexually active and is in a monogamous relationship. She describes no pain or soreness just an offensive smelling discharge. After examination and taking swabs for the second time she is diagnosed with bacterial vaginosis. Which of the following organisms is not likely to be the cause?
A 28-year-old woman attends her GP clinic for routine cervical screening. Liquid-based cytology (LBC) shows mild dyskaryosis. A repeat sample again shows mild dyskaryosis. What is the most appropriate management?
When assessing the fetal presenting part in labour it is important to know the anatomy of the pelvis. What are the bony landmarks of the pelvic outlet?
A 26 year old undergoes potassium-titanyl-phosphate (KTP) laser laparoscopic excision of endometriosis. Her postoperative haemoglobin is 8.1 g/dL. Six hours postoperatively she complains of increased umbilical swelling, abdominal pain and shortness of breath and she appears pale. A repeat full blood count now shows a haemoglobin count of 6.5 g/dL. What are the most appropriate steps you should take next?
A 54-year-old woman presents to her GP with a 1-year history of bloating, early satiety and occasional crampy pelvic pain. She was diagnosed a year ago with irritable bowel syndrome (IBS). A serum CA 125 is 62 IU/mL (normal range <36 IU/mL). What is the most appropriate management?
A 24-year-old woman is admitted to the gynaecology ward with a 4-day history of severe hyperemesis gravidarum. She has been unable to tolerate food or fluid orally for 2 days. On the second day of admission she develops signs of a severe pneumonia. This is presumed to be a hospital-acquired infection. She deteriorates rapidly. An arterial blood gas shows: pH 7.68, PO2 10.0 kPa, PCO2 4.26 kPa, HCO3 32 mmol/L, K+ 1.9 mmol/L, Lactate 1.2 mmol/L. What is the most accurate description of the acid-base disorder?
A 61-year-old woman has recently been diagnosed with a stage 1a endometrial carcinoma. She has had four children, she has mild utero-vaginal prolapse and she has never been operated on. She needs to have surgery. You see her in clinic and talk about the different operations available to her. Which is the most appropriate operation?
A 58-year-old woman presents to the clinic with post-menopausal bleeding. A pipelle biopsy confirms adenocarcinoma of the endometrium. Further imaging of the pelvis shows that there is spread of the tumour outside of the uterus into the left adnexa. There is no other spread. What is the most likely stage of the tumour?
A 65-year-old woman is referred by her GP to the gynaecology clinic with increasing bloating and a raised CA 125 level. A CT scan shows an irregular, enlarged left ovary and several well-circumscribed nodular lesions in the liver and on the omentum which are highly suspicious for metastatic ovarian cancer. What is the most appropriate treatment regimen?
A 62-year-old woman presents to accident and emergency with shortness of breath. Examination reveals reduced breath sounds and a swollen, distended abdomen. Chest x-ray demonstrates a left-sided pleural effusion. On further questioning the woman has had a poor appetite for the last 6 months and recently had some vaginal bleeding. An ultrasound revealed large quantities of ascites, which were drained. Analysis of the ascites shows a high protein content. What is the most likely diagnosis?
A 28-year-old woman attends accident and emergency unable to walk because she is so faint. She has had heavy vaginal bleeding for 4 hours since she engaged in sexual intercourse with a new partner, which she described as ‘rough and very painful’. She is still bleeding and cannot tolerate vaginal examination due to the pain. A point-of-care haemoglobin estimation is 6.4 g/dL and she is haemodynamically unstable. What is the most appropriate management?
A 64-year-old woman with asthma is admitted to the ward prior to an elective vaginal hysterectomy for symptomatic uterine prolapse. Her medications include Seretide (fluticasone/salmeterol 500/50) four times daily and oral prednisolone 20 mg twice daily. What is the most important peri-operative consideration?
A woman is undergoing surgery to enhance the cosmetic appearance of her labia. A bleeding vessel is encountered at the labia majora which cannot be controlled through pressure alone. The surgeon believes it to be a branch of the posterior labial artery. The posterior labial artery is a branch of which artery?
Following surgery to place a tension-free transobturator tape for stress incontinence, a 54-year-old woman loses some sensation in part of her labia anterior to the anus. Damage has most likely been caused to which nerve?
A 54-year-old woman with a history of significant ischaemic heart disease undergoes vaginal hysterectomy for symptomatic uterine prolapse. She develops significant surgical site bleeding which is repaired at reoperation the same day. Her postoperative haemoglobin is 6.4 g/dL. Later the same day she develops chest pain. Her observations, blood gas and cardiac enzymes are within normal limits. An electrocardiogram (ECG) shows sinus rhythm without ST changes. She is charted for thromboprophylaxis. What is the most likely cause of the chest pain?
A 46-year-old woman is returned to the ward from the recovery room following a routine vaginal hysterectomy for heavy periods and prolapse. The estimated blood loss at operation was 200 mL. Two hours later the ward sister becomes concerned that her urine output is low and calls the doctor. Her observations show: pulse 115 bpm, BP 90/62 mmHg, temperature 37.1°C. What are the most appropriate next steps in her management?
The peritoneal lining drapes over the pelvic viscera and forms the part of the peritoneal cavity. Which is the most inferior extent of the peritoneal cavity?
A 74-year-old woman has an annual health check up with her private insurer. They arrange an ultrasound scan that shows a cyst on her right ovary. It is multiloculated and has solid components. She is post-menopausal and otherwise well. A doctor has sent for a CA 125 which comes back as 120 U/mL. What is her risk of malignancy index score (RMI)?
A 21 year old comes to the clinic with a history of intermenstrual bleeding for the last 6 weeks. She has regular periods and does not experience post-coital bleeding. She is not on the oral contraceptive pill and has no other past medical history. What is the most appropriate first line investigation?
Two days after undergoing posterior exenteration for recurrence of cervical adenocarcinoma a 53-year-old woman develops a tachypnoea, tachycardia of 125 bpm and a fever of 39°C. Blood cultures have grown methicillin-resistant Staphylococcus aureus (MRSA). She requires intravenous vasopressors. What is the most appropriate diagnosis?
A 60-year-old woman is undergoing abdominal hysterectomy for a fibroid uterus. During suture ligation of the right uterine pedicle, iatrogenic injury to the ureter is confirmed. Which of the following statements is correct?
A 57-year-old woman has been referred by her GP under the 2-week suspected cancer referral approach with vaginal bleeding. She has been post-menopausal for the last 4 years and she has been taking Elleste Duet to treat her vasomotor symptoms. Two weeks ago, after reading about the risks associated with hormone replacement therapy (HRT) she stopped taking any medication. This is the first unscheduled bleeding she has ever had. She had a normal smear 2 years ago and is otherwise well. What would be your first line investigation?
A 39-year-old woman attends the gynaecology clinic complaining of long-standing pelvic pain. Routine bimanual examination and abdominal ultrasonography do not detect any abnormality. At diagnostic laparoscopy, multiple tiny dark brown nodular lesions are noted covering the surface of the uterus, tubes and left ovary, as well as in the Pouch of Douglas. Which finding is most likely from histological examination of the excised lesions?
A 21-year-old woman with dysmenorrhorea, dyspareunia and dyschezia has been scheduled for a laparoscopy to investigate possible endometriosis. You are asking for her consent and you describe the risks of laparoscopy, which include bleeding and damage to blood vessels, viscera and nerves. Which of the following is not at risk when inserting a lateral port?
A 14-year-old girl comes to see you as she has not had her periods yet. You note that her breasts are stage II and her nipples are set lateral to the mid-clavicular line. She has no pubic hair. Her weight is on the 50th centile but height is on the 9th centile. Her parents are both of average height. What is the most likely diagnosis?
A 2-year-old child is referred to you by the GP because he has not started walking. His mother says that he can stand but cries to be picked up or sits down shortly. His older sister was walking by 14 months. You note that he is talking well with short two to three word phrases. He is able to build a tower of six blocks. What is your management plan?
You see a boy in outpatients whose parents are concerned he is not talking yet. You do a developmental assessment and find he is walking well and able to build a tower of three blocks. He will scribble but does not copy your circle. He is able to identify his nose, mouth, eyes and ears as well as point to mummy and daddy. You do not hear him say anything but his parents say he will say a few single words at home such as mummy, daddy, cup and cat. He is a happy, alert child. Parents report him to be starting to feed himself with a spoon and they have just started potty training but he is still in nappies. What is the child’s most likely age?
A one-and-a-half-year-old Caucasian child is referred to paediatrics for failure to thrive. On examination he is a clean, well-dressed child who is quite quiet and withdrawn. He is pale and looks thin with wasted buttocks. His examination is otherwise unremarkable. What is the most likely cause of this child’s growth failure? His growth chart shows good growth along the 50th centile until 6 months followed by weight down to the 9th, height down to 25th and head circumference now starting to falter at 1.5 years.
A 12-year-old boy presents to his GP with left-sided unilateral breast development stage III. He is very upset as he is being bullied at school. His mother is worried as her friend’s sister has just been diagnosed with breast cancer and wants to know if he could have breast cancer? What is the management?
You see a baby for the first baby check at 6 weeks. Mum reports no problems and he is feeding well. On examination you are unable to palpate the testicles on ether side and do not feel any lumps in the groin area. He has a normal penis with no hypospadias and the anus is patent. He is otherwise a normal baby on examination. What is the most important diagnosis to rule out?
You see an 8-year-old boy in accident and emergency who fell off his bike 3 days ago and scraped his left calf. The cuts are now angry, red and painful. You note he is a big boy and plot his growth: his weight is on the 99th centile and height is on the 75th centile. You note mild gynaecomastia and stretch marks on his abdomen which are normal skin colour. His past medical history is unremarkable except for mild asthma. What is the most likely cause of his large size?
A 16-year-old boy is brought to the GP by his parents. They are concerned he is the shortest boy in his class. He is otherwise well. His height and weight are on the 9th centile. His father plots on the 75th centile and his mother on the 50th centile for adult height. On examination, his testicular volume is 8 mL, he has some fine pubic and axillary hair. The rest of the physical examination is normal. On further questioning you elicit from his father that he was a late bloomer and did not reach his full height until he was at university. What is the most likely cause of the boy’s short stature?
A mother comes to see you with her 2-year-old daughter, Stacey, out of frustration that her daughter is so ill behaved. She does not know how to make her listen and is worried that she is going to get hurt. Yesterday she ran ahead and did not stop when her mother called to her. She ran into the street and was hit by a cyclist, but fortunately he was OK and Stacey had only had a few cuts and scrapes and seems alright! On questioning you hear other stories of a naughty child. She is active and eats well, feeding herself a lot now, but her mother does say she gets frequent coughs and colds. Her mother says that Stacey only says about 5–10 words and only she can understand what Stacey says. What is the best next management?
An older mother books in to see you after attending the health visitor for a weight check at 2 months for her first child. She and her husband have had a hard time coming to terms with their daughter’s diagnosis of Down’s syndrome. She is relieved that the appointment with the cardiologists went well and the heart is normal. However they have a lot of trouble getting her to take the whole bottle, she was slow to regain her birth weight and looking at the plotted weight yesterday she is not growing along her birth centile and the mother is worried she is not doing a good enough job. She is not vomiting except for small possets after feeds, is passing urine and opening her bowels. The red book growth chart shows the weight to be falling off centiles. What is the most appropriate management?
Which child should be moved to the resuscitation area for urgent management in accident and emergency?
A 4-year-old child has been losing weight recently and has been vomiting for the past 24 hours, unable to eat anything. His mother has brought him into accident and emergency out of concern as he seems confused. The triage nurse has taken him to the resuscitation room and asked for your help. On examination he is drowsy, has a heart rate of 150, respiratory rate of 60 and a central capillary refill of 5 seconds. He has subcostal recessions and good air entry bilaterally with no added sounds. He moans when you examine his abdomen but there are no masses. You put in a canula and take bloods. The venous blood gas shows: pH 7.12, PCO2 2.3 kPa, PO2 6.7 kPa, HCO3– 15.3 mmol/L, BE –8.6, Glucose 32.4 mmol/L. What is the most likely diagnosis and what is the first management step?
An 8 year old known asthmatic is brought into accident and emergency by ambulance as a ‘blue call’. He has been unwell with an upper respiratory tract infection for the past 2 days. For the past 24 hours his parents have given him 10 puffs of salbutamol every 4 hours, his last dose being 90 minutes ago. The ambulance staff have given him a nebulizer but he remains agitated with a heart rate of 155, respiratory rate of 44 and sub/intercostal recessions and on auscultation there is little air movement heard bilaterally. Saturations in air are 85 per cent. He is started on ‘back to back’ nebulizers with high flow oxygen. How severe is his asthma exacerbation and what other bedside test would support this?
The accident and emergency triage nurse asks you to look at a 3-year-old child with a short history of waking up this morning unwell with a cough and fever. She looks unwell, heart rate is 165, respiratory rate 56, saturations of 96 per cent in air, temperature of 39.3°C and central capillary refill of 4 seconds. She has a mild headache but no photophobia or neck stiffness and you notice a faint macular rash on her torso and wonder if one spot is non-blanching. You ask the triage nurse to move her to the resuscitation area and call your senior to review her. Fifteen minutes later your senior arrives and the spot you saw on the abdomen is now non-blanching and there is another spot on her knee. What are the three most important things to give her immediately?
A 9-year-old boy is brought in by ambulance having been hit by a car while playing football in the street. You have been assigned to do the primary survey in resus when the ambulance arrives. The patient is receiving oxygen, crying for his mummy and holding his right arm, but able to move over from the stretcher to the bed when asked. Which is the correct examination procedure?
A 6-year-old boy with a history of anaphylaxis to peanuts is brought in by ambulance unconscious. He was attending a children’s birthday party. His mother says there was a bowl full of candy and he may have eaten a Snickers bar but she is not sure and she did not have his EpiPen with her. His face and lips are swollen and erythematous, he is still breathing but weakly and there is wheeze. His pulse is tachycardic and thready. Which type of shock is this?
A 13 month old is brought in having had a blue floppy episode at home lasting 1 minute. While you are taking a history from the mother, you notice the baby has gone blue again and seems to be unconscious in her arms. You call for help and place the baby on the examination table. There is no obvious work of breathing. The nurses bring the crash trolley and give you a bag valve mask, which they are connecting to the oxygen. You give two inflation breaths but do not see the chest rise. You reposition the air way and this time the breaths go in. You feel for a pulse and there is none. When asked to do CPR the nurse asks for direction on how many breaths and compressions you both need to do.
A 10-year-old child is brought in by ambulance with seizure activity. His mother reports it starting 30 minutes ago in his right arm and quickly became generalized tonic clonic jerking. She gave him his buccal midazolam after the first 5 minutes and called an ambulance when he did not respond after another 5 minutes. The ambulance crew gave him rectal diazepam on arrival at 15 minutes into the seizure. He is receiving high flow oxygen via a face mask and continues to convulse. The mother tells you that he was weaned from his long-term seizure medication, phenytoin, 2 weeks ago and that he has had a cold for the past 2 days. What is the next step in management?
A 3-year-old boy is brought in by ambulance fitting. You are assigned to get the history from the father. Harry is normally fit and well with no significant past medical history or allergies. He is up to date with his immunizations and has been growing and developing normally. His behaviour has been difficult for the past 2 weeks since the birth of his little sister. Mum has been unwell as she developed HELLP syndrome and was in hospital for a week following the delivery. Yesterday, he was quite unwell with a tummy bug, vomiting and had black diarrhoea. That evening they found a mess he had made in the bathroom with all of his mum’s things strewn over the floor including her tablets from the hospital. By that time, Harry was getting better so they did not think anything of it. Today he has been acting strangely and has been difficult to understand, he then became lethargic at about 4 pm and started fitting 15 minutes ago. What is the most likely diagnosis?
A 6-year-old boy with a history of asthma and eczema is brought in to accident and emergency from a local restaurant. He is on high flow facial oxygen with significant facial oedema and generalized erythema. On auscultation there is widespread wheeze for which the ambulance crew gave a salbutamol nebulizer. What is the next step in management?
A newborn baby is born to non-consanguineous parents. She is noted to have puffy feet on her 1st day check. She weighs 2.0 kg with widely spaced nipples and absent femoral pulses. You have asked your registrar to review her as you think she may have Turner’s syndrome. She agrees and asks you to send blood tests for karyotyping. Which is the chromosomal diagnosis of Turner’s syndrome?
A 15-year-old boy was diagnosed with Down’s syndrome at birth. He is short for his age, had cardiac surgery as a baby, has treatment for hypothyroidism and now attends mainstream school with some support. His parents are enquiring now about what complications he faces. Which of these is not a recognized complication of Down’s syndrome?
A baby is born and you are asked to do the baby check at 6 hours post-natal age. You go to see the baby and mum states that he has not yet had a feed. You advise they stay in hospital until the feeding is established. This is the first child of non-consanguineous parents. On day 4 when you review the baby he has still not had an adequate intake, has lost over 10 per cent in birth weight and is markedly hypotonic. Your consultant asks you to request genetic testing for Prader–Will syndrome. What is the inheritance of Prader–Willi syndrome?
A 5-day-old baby who is formula fed is on the neonatal unit being treated for sepsis secondary to an Escherichia coli urinary tract infection. He has been on antibiotics for 5 days. He is still unwell and vomiting. The parents are consanguineous and this is their first child. He has had repeat blood and urine cultures taken. Urine reducing substances are positive. What is the most likely underlying diagnosis?
A 10-year-old boy is brought to the GP with tall stature. He is taller than his peers at school. His arm span is greater than his height, he has long, thin fingers, scoliosis and pectus excavatum. He is also concerned that he gets short of breath at school during PE lessons. You refer him for an echocardiogram and chest x-ray. You make a clinical diagnosis of Marfan’s syndrome. What is the inheritance of Marfan’s syndrome?
A pregnant woman seeks advice from you regarding her condition and its impact on the pregnancy and risk to the baby. She has phenylketonuria (PKU) and has been on a phenylalanine-free diet for life. She was told that it was very important during her pregnancy to be compliant with this diet. She would like to know how the baby will be tested for the condition as she is aware that is an inherited condition. What is the initial investigation you will advise?
You see an 18-year-old boy who is the first child of his African parents and was born in Kenya before moving to the UK 1 year ago. He has white skin and pink irises. He was diagnosed with oculocutaneous albinism at birth. He has difficulty with his sight but has recently developed a skin lesion on his face. His mother has brought him to his GP as it has recently started to increase in size. On examination you note is an elevated, 3 cm diameter lump on the left of his nose. It has irregular edges, is firm and immobile and pigmented in areas. What is the likely diagnosis?
A 20-month-old boy has been referred due to delayed walking. On further questioning you establish he has no difficulty feeding, had head control at 3 months of age, and sat up by 8 months. He has been crawling for the last 8 months, but he does not pull to stand or walk with support. He has no dysmorphic features. There is no known family history of muscle problems. His mother has no myotonia. His mother is very concerned and asks you what is wrong. What is the most likely diagnosis?
You are asked to see a 3-day-old baby on the post-natal ward. The baby was born at term and is the first child of consanguineous parents. The baby is drowsy and vomiting, with no fever, rash or diarrhoea. On examination, the baby is noted to have ambiguous genitalia. You do some blood tests: white cell count 5 × 109/L, C-reactive protein 2 mg/L, Na+ 125 mmol/L, K+ 8 mmol/L, glucose 1.7 mmol/L. 17-OH level progesterone is low. You make a diagnosis of congenital adrenal hyperplasia. What is the best initial management plan?
A 10-year-old boy is brought to the paediatric outpatient department for a review of his height. He was found to be on the 0.4th centile and his mid-parental height is the 98th centile. He also has widely spaced nipples, wide carrying angle, hypogonadism, pulmonary stenosis and developmental delay. What is the most likely diagnosis?
A 2-week-old baby was referred to the prolonged jaundice clinic by the community midwife. The pregnancy was unremarkable, and she was born at term with no antenatal abnormalities on ultrasound (US) scans or blood serology. She is now 17 days old and has been jaundiced since day 5 of life and never required phototherapy. She is breastfed and feeds 3-hourly for 20–25 minutes. She is afebrile and not lethargic. Her mother reports that the stools are pale and she has dark coloured urine. The bilirubin is 300 μmol/L, and conjugated bilirubin 100 μmol/L. What is the most important diagnosis to exclude?
A 1-day-old baby is on the post-natal ward. You are asked to review her as she is febrile and lethargic. On examination she is tachycardic, has a capillary refill time of 3 seconds centrally and reduced urine output. Her blood culture 24 hours later grows Gram-positive cocci. Which is the most likely causative organism?
A preterm baby is born at 25 + 6 weeks gestation. He is delivered by caesarean section due to maternal pre-eclampsia. He is intubated at birth and given surfactant via the endotracheal tube. He is ventilated and commenced on IV dextrose. After 4 hours of age he has increased work of breathing, with intercostal and subcostal recession and a respiratory rate of 60/min. A chest x-ray shows a ground glass pattern in both lung fields. He has no audible murmur. He is afebrile. You diagnose respiratory distress syndrome. What is the aetiological factor responsible for respiratory distress syndrome?
A preterm baby is now 25 + 7 weeks corrected gestation. He is on the neonatal unit being cared for while his mother recovers on ITU after he was born secondary to an eclamptic seizure. He has been receiving formula milk as the parents have not consented to donor breast milk. He has been having bilious aspirates from his nasogastric tube and today his abdomen in very distended and tense. He has had one episode of bloody stools. You are going to treat him for nectrotizing enterocolitis (NEC). What is the best initial management plan?
A 3-day-old baby is seen by the midwife for a routine post-natal review. She notices that he is very floppy and his mother has raised concerns about his poor feeding. He has a protruding tongue, epicanthic folds, low set ears and sandal gap toes. She explains to the parents she thinks he may have Down’s syndrome and refers him to the paediatrician. What is the diagnostic test for Down’s syndrome?
A baby is born by emergency caesarean section due to fetal tachycardia. His delivery was uneventful and you are asked to see him 5 hours later on the post-natal ward. He has just taken his first feed and has been coughing and spluttering since. He had an episode with blue lips transiently and this has now improved; his oxygen saturations are 97 per cent in air and he is apyrexial. On examination, you note other features including vertebral and limb abnormalities, imperforate anus, pansystolic murmur at the lower left sternal edge and renal anomalies noted on antenatal scans. What is the most likely cause for his coughing episode?
A term baby is awaiting his discharge check when you are called to see him at 10 hours of age. His mother reports that he has turned a dusky colour and is not as alert as he has been. On examination he has central cyanosis, pulse 150 bpm regular, and both brachial and femoral pulses are palpable. He has normal heart sounds with no murmur. His oxygen saturations are 65 per cent in air. What is the most likely underlying diagnosis?
A 12-hour-old baby on the post-natal ward has just had a seizure lasting 2 minutes. It resolved spontaneously and was generalized in nature. Her mother had gestational diabetes and poor glucose control in pregnancy. The baby’s birth weight was 5 kg. There were no abnormalities noted on antenatal US scans or maternal serology. On examination she has no dysmorphic features and handles well. What initial blood tests would you do for the baby?
A pregnant woman is admitted to the labour ward for an elective caesarean section at 38 weeks for her baby who had an antenatal diagnosis of gastroschisis. The paediatric team are called to attend the delivery. The baby is born in good condition with no resuscitation required. He is taken to the neonatal unit for further care. Which of these is a complication of gastroschisis?
You are called to see a baby who has just been born at 39 weeks’ gestation, as the midwife thinks he is small and should be admitted to the neonatal unit for his care. You review the baby. His weight is 1.8 kg, below the 0.4th centile and his head circumference is 35 cm – 50th centile. He has no dysmorphic features. Which is the most likely cause of this IUGR?
A mother brings her 4-week-old baby to see you for the third time. He was born at term by normal vaginal delivery with no complications. You started him on anti-reflux medicine last week but it has not helped. He is now vomiting his whole feeds and is becoming lethargic and passing less urine and stool. His mother says he is hungry even after he vomits. The practice nurse has weighed him and he has lost 200 g since last week. His mother was breastfeeding him while waiting to be seen and as you go to examine him, the baby has a large milky vomit, which cascades over the clinic floor. What is the most likely diagnosis?
A 15-month-old girl has come to see you with her father. The family are worried that she has had diarrhoea for more than a month, occasional vomiting and is losing weight. She used to be a happy interactive baby but now seems lethargic and miserable most of the time. She has no significant past medical history, the rest of the family are well and there is no history of travel. Her mother has well-controlled type 1 diabetes. The child’s weight at 6 months in the personal child health record (‘red book’) was on the 50th centile but she is now just below the 9th. What is the most likely diagnosis?
A 13-year-boy is brought to see you as he has recently been complaining of abdominal pain and is increasingly tired. On examination you note some early clubbing and erythematous palms. His conjunctivae look pale. He has one or two spider naevi on his chest. His abdomen is soft with mild tenderness in the epigastrium and right upper quadrant. The liver is palpable at 1 cm and you feel the splenic tip. He has normal bowel sounds and no bruits. On slit lamp examination of his eyes, an amber ring is noted around the cornea. What is the most likely diagnosis?
An 8-year-old girl is brought to see you, having not opened her bowels in 8 days. She complains of hard painful stools and recurrent abdominal pain for the past 6 months but no vomiting. Her mother thinks that she is avoiding going to the toilet and reports that she has always been a bit irregular opening her bowels, averaging about twice a week. In her past medical history, she passed meconium on day 1 of life and has had no significant medical problems. On examination she is a well-looking, normally grown child. Her abdomen is soft with a palpable indentable mass in the left iliac fossa. The anus is normal, as are her lower limbs. What is the first step in management?
A 2-week-old baby is brought to accident and emergency by his parents because he has been intermittently inconsolable for the past 12 hours. He does not want to breastfeed and has vomited. The parents think his tummy is upset as he keeps drawing up his legs. He was born at term by normal vaginal delivery with no problems. On examination the abdomen is distended and tense. He is crying and there is a firm swelling in the right groin area. You can hear active bowel sounds. What is the most likely diagnosis?
A 5-year-old girl is brought to accident and emergency with a 24-hour history of vomiting and diarrhoea and now her eyes and skin have gone very yellow. She has been taking oral rehydration salts and is still passing urine. She is normally healthy and there is no family history of jaundice. On examination her heart rate is 130 and respiratory rate is 26. She is alert, warm and well perfused. The chest is clear, heart sounds are normal and the abdomen is soft with a 2 cm liver edge. What should the management be?
A 15 year old with well-controlled type 1 diabetes presents with frank haematemesis. Her blood tests in accident and emergency show: pH 7.37, glucose 18.3 mmol/L, haemoglobin 12.3 g/dL, white cell count 5.3×109/L, neutrophils 2.1×109/L, platelets 165×109/L, Na+ 135 mmol/L, K+ 3.5 mmol/L, urea 5.0 mmol/L, creatinine 83 μmol/L, alanine transaminase 740 IU/l, bilisubin 96 μmol/L, alkaline phosphatase 102 IU/l, and albumin 25 g/L. Further investigations once she is stable on the ward show hepatitis B surface antigen negative, anti-hepatitis C virus negative, anti-nuclear antibody (ANA) 1:320 and anti-smooth muscle antibodies are positive. What is the most likely diagnosis?
An 18-month-old child is brought into accident and emergency with a 2-day history of vomiting, abdominal pain and fever. Which of the following is an unlikely cause of this clinical picture?
A 13 month old is referred up to her local district general accident and emergency by a GP who is concerned she has intussusception following an 18-hour history of fever, vomiting and intermittent colicky screaming. A kind radiologist agreed to do an urgent ultrasound which shows an area of invaginated bowel in the right side of the colon. What is the most appropriate management?
Which of the following is not a cause of PR bleeding?
A mother brings her 2 year old to see you. She is very worried that he always has diarrhoea or loose stools. He eats a normal diet, and no particular foods seem to upset him but he often still has bits of vegetables or food he has eaten visible in the stool. She thinks he is losing weight and he is starting to potty train, so she is concerned this will affect his ability to anticipate needing the toilet. On examination he is an alert and well-looking child with a normal capillary refill, heart and respiratory rate. His abdomen is soft with no masses, there is no evidence of wasting and his weight and height are following the 50th centile. What is the most appropriate management?
A 10-year-old boy presents with recurrent mouth ulcers, abdominal pain, distension and frequent episodes of diarrhoea with mucus. He has been losing weight. On examination he is slim and plotting his growth shows a fall in weight from the 50th centile to below the 9th. His abdomen is soft with generalized discomfort on deep palpation but no masses are present. What is the most likely diagnosis?
A 15-year-old boy comes to see you, complaining of recurrent abdominal and back passage pain relieved by passage of diarrhoea. He is also complaining of low back and knee pain and last week there was blood mixed into his stool. He has been losing weight recently. On examination he is slim and looks pale. His abdomen is soft but tender in the left iliac fossa with no masses. What is the most likely diagnosis?
Ninety-nine per cent of healthy term infants will pass meconium within the first 24 hours of life and all should do so within 48 hours. Which of the following is not a cause of delayed meconium passage?
A 3-month-old baby is brought to accident and emergency because he has been vomiting and having diarrhoea for the past month. His mother breastfed him until he was 8 weeks old and he is now taking formula milk, 4–5 oz every 4 hours. On examination he is alert but fussy and looks thin. He has eczema on his face, neck and torso and the mother says this is new. The abdomen is soft, the genitalia are normal with a significant nappy rash and the anal margin is erythematous. You plot his growth in his red book and find that he was born on the 50th centile and was following that but now he is on the 25th centile for weight. What is the most likely diagnosis?
A 5-year-old boy was diagnosed with asthma aged 3 years. He presented to accident and emergency with shortness of breath, increased work of breathing and a 1-week history of coryzal illness and fever. On examination he is tachypnoeic 60/min, tachycardia 160 bpm and has minimal air entry bilaterally. He has intercostal recession, tracheal tug and is too breathless to complete a sentence. Oxygen saturation is 90 per cent in air. What is the initial management of this boy?
A 3-year-old child presents to the GP with a chronic cough for the last month. He had previously been fit and well since he suffered a severe pertussis infection when he was 1 month of age. He has subsequently been fully immunized but was noted to be on the 0.4th centile for height. What is the most likely cause for his cough?
A 15-year-old boy attends his GP with a week of cough productive of yellow sputum, fever to 39°C and chest pain on the right side of the chest on coughing. There is no history of foreign travel or unwell contacts. On examination there is reduced air entry in the right lower zone with crepitations and bronchial breathing. You diagnose a right-sided chest infection. What is the most likely causative organism?
A 4-year-old girl has recently moved to the area and is registering with you, her new GP. She has had a diagnosis of primary ciliary dyskinesia (PCD) made last week and the parents wish to know more about the complications. Which of the following is not a complication of PCD?
A 26 week, premature baby was born by emergency caesarean section due to maternal pre-eclampsia. He required ventilation until age 38 weeks corrected gestation and is still requiring oxygen to maintain his saturations. At 12 months of age he has poor vision and neurodevelopmental function, requires home oxygen and was admitted for a recent respiratory syncytial virus (RSV) bronchiolitis. What is the underlying diagnosis of his respiratory problems?
John is a 2-year-old boy whose mother has been concerned about a cough for the last 2 weeks which started out of the blue. He has been previously fit and well with no respiratory or cardiac problems from birth. There is no family history of illness. He is thriving and eating as normal, but has a persistent cough, recently productive of yellow and slight blood stained sputum. You suspect that John may have a pneumonia and lung collapse secondary to an inhaled foreign body. Which is the most likely location of this boy’s foreign body?
A couple who are known to both be carriers of cystic fibrosis ask to see you. They had genetic counselling but declined antenatal diagnostic testing and their baby has now been born and is ready to be discharged home. The parents are now keen to get the baby tested so that if treatment is required it can be initiated early on. What initial test do you suggest for the baby?
A 5-week-old baby was admitted today to the children’s ward with bronchiolitis. The nasopharyngeal aspirate identified respiratory syncitial virus. He was saturating to 96 per cent in air this morning and was feeding two-thirds of his usual amount of formula milk. You are asked to review him as his work of breathing is worsening now it is night time. He has nasal flaring, intercostal and subcostal recession, tachypnoea and crepitations and wheeze heard bilaterally. What do you expect his capillary blood gas to show?
Clara is a 14-year-old girl who was diagnosed with muscular dystrophy when she was younger. She now mobilizes in a wheelchair and other co-morbidities include a scoliosis and cardiomyopathy. She is being seen for her annual review in clinic. Which of these would best represent the respiratory complications of muscular dystrophy?
A 10-month-old baby boy is brought to accident and emergency with inconsolable crying. His mother says he is a miserable baby and even after feeding he does not settle. He has recently started to cruise around furniture, but is not yet walking. His crying has been worse today and both his parents had been awake all night due to his incessant crying. On examining the baby, you note that he is more upset when being handled and is a bit better when lying on his front. You do a chest x-ray which shows three posterior rib fractures; his mother states he fell down some steps yesterday. What is the likely diagnosis and appropriate management strategy?
What is the most common congenital heart defect?
Which of the following is not a presenting symptom or sign associated with congenital heart disease?
Which of the following is not a feature of an innocent murmur?
You are asked to see a 2-year-old child with difficulty in breathing, a runny nose and a barking cough. His mother tells you he had a heart defect repaired as a baby and he still has a murmur. On examination he has noisy breathing with mild subcostal recession. He is apyrexial with a respiratory rate of 44 breaths per minute and heart rate of 152 beats per minute; capillary refill is 1–2 seconds. His throat is red and the tonsils are enlarged with no exudate. On his chest you see a midline sternotomy scar with a drain scar and a right thoracotomy scar. On auscultation the lung fields are clear, but he has an ejection systolic murmur in the left upper sternal edge which radiates to the back. He does not have a gallop rhythm. There are transmitted upper airway sounds only on the lung fields and the abdomen is soft with no organomegaly. What is the most appropriate management?
A 3-day-old baby is brought to accident and emergency with acute respiratory distress. She is tachypnoeic, tachycardic, cyanosed and her capillary refill is 5 seconds centrally. You note she has a flat nasal bridge, down sloping palpebral fissures and epicanthic folds. On auscultation there is a loud ejection systolic murmur at the left sternal edge. What is the most likely diagnosis?
You are doing a baby check on the post-natal ward on a baby who is 23 hours old. His mother tells you that he is not feeding well. On examination he is unsettled with a respiratory rate of 76 and a heart rate of 182. You think his hands and feet look blue and there is a soft systolic murmur heard at the left upper sternal border. You ask the midwives to check his saturations which are 85 per cent in air and start some oxygen. You explain to the mother that he needs to be managed on the neonatal unit. What is the next step in your management?
A 7 year old with a 3-day history of upper respiratory tract infection is brought to accident and emergency by his mother because he suddenly went pale and sweaty and seems to be working hard to breath. The triage nurse calls you to see him urgently because his heart rate is 200 beats per minute. You take him round to the resuscitation area, give him oxygen and connect him to the cardiac monitor. The electrocardiogram (ECG) shows a narrow complex tachycardia with a rate of 180 beats per minute. He remains alert, with a respiratory rate of 40. What is the most appropriate initial diagnosis?
What is the first step in management?
A 2-year-old child is referred to hospital by the GP after his third visit that week; he now has a rash and the GP is worried he has meningitis. He has had a fever for 5 days up to 39.5°C or above every day and is not eating or drinking well. On examination, he has a temperature of 38.5°C, heart rate of 150, respiratory rate of 30 and is miserable. He has a blanching macular rash on his torso, swollen hands and feet, red eyes, red cracked lips, large tonsils with no pus, and a left-sided 2 cm × 3 cm cervical lymph node which is mobile. There is no photophobia or neck stiffness. His chest is clear with normal heart sounds and his abdomen is soft with a palpable liver edge. You note his BCG scar is inflamed. What is the most likely diagnosis?
A 14-year-old refugee from Afghanistan who has lived in the UK for 2 years comes to see you complaining of increasing fatigue and breathlessness on exertion. On examination she appears cyanosed and has bilateral basal fine crepitations and a soft pansystolic murmur with a displaced apex beat. She has never been in hospital and has no surgical scars. You urgently refer her for a cardiology review. What is the most likely diagnosis?
The major criteria for rheumatic fever include all of the following features except?
A 5-year-old child was admitted overnight awaiting surgical repair of a broken right ankle and was noted to have a raised blood pressure consistently above 130/90 mmHg despite adequate analgesia. On examination he has a plaster on his right foot and appears comfortable at rest. On auscultation there is a soft systolic murmur heard at the right upper sternal edge. His femoral pulse is difficult to find, but present bilaterally. When felt with the radial pulse, the impulse in the femoral pulse occurs slightly later. His abdomen is soft and there are no bruits heard. The blood pressure done in the right arm is 136/92 mmHg but the left arm gives a reading of 124/80. What is the most likely diagnosis?
Which of the following is not a feature of cardiac insufficiency?
Forty per cent of children with trisomy 21 have congenital heart defects. Which of the following is not associated with Down’s syndrome?
A 14-year-old girl was seen in accident and emergency following her third collapse this year and referred to cardiology for review of a low rumbling murmur heard at the left upper sternal edge. Her ECG in accident and emergency was normal. Her blood sugar was 5.3 mmol/L. Urea and electrolytes were normal. The most recent collapse occurred at school while waiting for exam results to be given out. Previously they occurred while watching a parade all afternoon standing in a crowded street, and at a party. On all three occasions she felt dizzy beforehand, was unconscious for less than 10 seconds and fully alert following the episode, but did feel nauseous. Her echocardiogram today is normal. What is the most likely diagnosis?
A 1-month-old baby attends accident and emergency with a 2-day history of fever to 38.8°C measured at the GP surgery. He has been vomiting, with no diarrhoea, rash, cough or coryza. A clean catch urine has leukocytes +++ and ketones, no nitrites, blood or protein. An urgent microscopy shows >200 cells/μL white cells. What is the most appropriate course of action?
A 5-year-old boy presents to his GP with a 3-day history of puffy eyes. He has been unwell with a coryzal illness for the last week. His mother states he has had no new medications and no hayfever, allergies or asthma. On further examination he has generalized oedema and scrotal oedema. He is tachycardiac and has cool peripheries, no skin rashes or erythema. What is the most likely diagnosis?
A 12-year-old girl presents to her GP with a UTI. She has no past medical history of note and is not taking any medication. On testing her routine observations, her blood pressure was 140/90 mmHg with a manual sphygmomanometer. You are concerned this may be high for her age. She has no headaches, visual disturbance, vomiting, chest pain, dyspnoea or neurological signs. What is your next course of action?
A 6-year-old girl presents to hospital with a large right-sided abdominal mass. It does not cross the midline. On further questioning she has had macroscopic haematuria and weight loss of 4 kg over the last 4 months. She has reduced appetite and lethargy. Her blood pressure is 125/73 mmHg, heart rate 120 bpm. Which of the following is not a complication of this malignancy?
James is an 8-year-old boy who recently attended accident and emergency with a swollen left ankle. He had an x-ray and was discharged home and told there was no fracture. He has now developed a dark purple rash on his legs, which does not disappear with a glass pressed on. He was brought back to the department today vomiting, with abdominal pain. His observations and urine dipstick are all within normal limits. What is the most likely diagnosis?
A 7-year-old boy presented to accident and emergency with diarrhoea and vomiting for the past week. He had no history of foreign travel and but had been to a zoo recently on a school trip. He was discharged home, after providing a stool sample, with rehydration advice as he was less than 5 per cent dehydrated and tolerating oral fluids. The stool had grown ‘Escherichia coli 0157’ which was phoned from the microbiology laboratory to the on-call doctor 48 hours later. What is the most serious complication?
A 5-year-old girl was brought to hospital at midnight by her mother with 5 per cent partial thickness burns to her chest and abdomen. Her mother states that she pulled on the kettle at 2 pm and the boiling water scalded her. On examination she is tachycardic, and drowsy with cool peripheries. Her initial blood tests: sodium 150 mmol/L, potassium 7.8 mmol/L, urea 10.2 mmol/L, creatinine 104 μmol/L, haemoglobin 14 g/dL. What is the most likely aetiological factor to account for these results?
A 10-day-old baby boy was brought to accident and emergency with a distended abdomen. On questioning, he was born at term with no antenatal concerns. Until 2 days ago he had been feeding well and not vomiting, he had been wetting nappies, but mother has not witnessed a good urinary stream. On examining the child, you find a mass, dull to percussion, arising out of the pelvis, and he has had no wet nappies for the last day. You suspect he may have posterior urethral valves. Which one test will help to diagnose this underlying condition?
A 12-year-old boy who was born with multicystic dysplastic kidneys. He had a renal transplant when he was 7 years old due to chronic kidney disease stage V after having peritoneal dialysis for 1 year. Which of the following would you not expect him to be taking?
An 11-year-old girl presents to the out of hours GP while on holiday in England with abdominal pain. She tells you she has polycystic kidney disease which was diagnosed early in life. She has bilateral palpable kidneys and hepatosplenomegaly, with visible distended veins on the abdomen and ascites. Abdominal ultrasound shows liver fibrosis. What is the inheritance of this condition?
An 11-year-old girl was brought to accident and emergency in December with pain in her left leg. She is known to have sickle cell disease and her baseline haemoglobin is 7.0 g/dL. She has been admitted in the past with painful leg and chest crises. She has a cough and coryza. Today her blood results show: haemoglobin 6.8, white cell count (WCC) 12 × 109/L, platelets 209 × 109/L, C-reactive protein (CRP) 20 mg/L. What is not part of the appropriate initial management?
A 4-year-old boy is brought to accident and emergency with a limp for 1 day. He was unhappy to weight bear on his right leg. He had been with his grandparents all day and his mother brought him to hospital when she returned from work that evening. He was afebrile with a heart rate of 110 bpm but had had a cold last week. Mum reports no history of trauma. What is the most important diagnosis to exclude?
A 14-year-old girl was diagnosed with idiopathic thrombocytopenic purpura (ITP) last week after she attended the children’s assessment unit with recurrent epistaxis. She had a platelet count of 16 × 109/L last week and now re-presents to accident and emergency with further episodes of epistaxis, haematemesis and petechiae. She had a heart rate of 110 bpm and her blood pressure is 100/70 mmHg. What is the next best management step?
A 14-year-old girl went to her GP with a sore throat and cervical lymphadenopathy. She had a blood test done and you are called later that day with results. Haemoglobin 6.0 g/dL, WCC 230 × 109/L, neutrophils 0.9 × 109/L, platelets 77 × 109/L; blood film showed blasts and Auer rods. What is the most important management priority for this child in the first 24 hours from diagnosis?
A 2-year-old boy is admitted to the paediatric ward with a swollen, painful left knee. He has been afebrile and has a history of minor trauma to his knee earlier today. His mother is a haemophilia carrier and his father is not affected. You are keen to rule out haemophilia in this child. Which two clotting factors should you test for?
A 20-year-old man presents to the infectious diseases department with a large 7 cm × 8 cm swollen painful lump in the left anterior triangle of his neck. He has night sweats, 10 kg weight loss and a dry cough for the last month. He was treated with surgery and radiotherapy for a high grade astrocytoma when he was 8 years old. Which of the following is not a recognized complication of his childhood condition and its treatment?
A 12-year-old girl has been seeing her GP for the last year with heavy periods and had suffered with bleeding gums when she was younger. She is otherwise well and lives with her adoptive parents who now have parental responsibility. Her coagulation tests reveal normal prothrombin time (PT) and activated partial thromboplastin time (APTT), low factor VIII, low von Willibrand factor (vWF), abnormal platelet aggregation and increased bleeding time. What is the likely inheritance of her condition?
A 4-year-old girl has just returned from holiday in France where she visited a petting farm. She has had diarrhoea for 2 days, and her mother noticed fresh red blood mixed with the stools. She has also been vomiting. On admission to hospital her blood tests showed: Hb 5 g/dL, WCC 15 × 109/L, platelets 55 × 109/L, urea 19 mmol/L, creatinine 110 μmol/L. Her stool culture is pending. What is the most likely diagnosis?
An 18-month-old boy presented to the GP with a history of eating soil. He had been in the garden this afternoon as his mother put the washing out. She found him eating the soil and took him straight inside. On examination, he is well and alert but has pale conjunctivae. He is not tachycardic or tachypnoeic. His diet consists of predominantly of breast milk. What is the most likely result of his haemoglobin and haematinics?
A 9-month-old boy presented to his GP with lethargy and a prominent forehead. He is pale on examination and has yellow sclerae. He is the first child of his non-consanguineous parents. His haemoglobin is 6.5 g/dL, WCC 5.0 × 109/L, platelets 300 × 109/L. His blood film shows evidence of haemolysis, no spherocytes, no sickle cells and a good reticulocyte count. Direct antiglobulin test (DAT) is negative. What is the most likely diagnosis?
A 3-year-old is brought into accident and emergency on a Monday morning because she has developed several bruises on her buttocks, left leg and right arm. She is seen with her nanny who reports finding the bruises when she was getting her dressed this morning. Recently the girl has not been herself. She has had several colds over the past 2 months and has been more lethargic lately. The nanny is worried she is losing weight. On examination she appears withdrawn, pale and has a bruise on the left buttock which is 5 cm × 8 cm. She has three other bruises on her left leg and right arm which are of varying colours. She also has some fine petechiae on her neck and cheeks. She has a runny nose and a cough but the chest is clear. What is the most likely diagnosis?
On the day 1 baby check a mother is very concerned about a rash on her baby’s face. Over the right eye, forehead and temple there is a pink-red, flat area of erythema. He is opening the eye, and his eye movements seem intact. The child’s observations and rest of the examination are normal. What should you tell the mother?
A mother brings her 6-month-old, formula-fed baby to see the GP complaining that the olive oil she is using is not helping his persistent cradle cap and worsening rash on his face and arms. On examination he has extensive cradle cap and eczematous changes on his cheeks, neck, chest and arms. The neck skin creases are red and oozing with yellow crusts. He is miserable and feels warm to touch. What is the most appropriate management?
A 4-year-old is brought into accident and emergency by very anxious parents. She has had a bad cough which makes her vomit and a fever for 2 days. She has now developed a rash on her face which does not pass the ‘glass test’, in that the spots are still visible when a glass is pressed against the skin. On examination she is alert and comfortable at rest, with fine petechiae on her cheeks and neck which are non-blanching. She has red, enlarged tonsils without pus and the chest is clear. What is the most likely cause of her rash?
A 2 year old was seen in accident and emergency by the senior house officer with a short history of fever, malaise and now vomiting. She had a blanching rash on her arms and abdomen. She looked unwell but had no clear focus for her fever. She was tachypnoeic but her chest was clear. A urine sample was requested which showed a trace of leukocytes and two plus of ketones. Forty-five minutes later the paediatric registrar came to review the child who appears lethargic with a capillary refill centrally of 6 seconds and the rash on her abdomen is now non-blanching. What is the most likely diagnosis?
A 2 year old is brought in by ambulance after pulling a pot of boiling water off the stove down on top of himself. He has significant burns to the whole of his face, torso and right arm. Estimate the percentage body surface area affected.
Which of the following is not a cause of erythema nodosum?
A 3 year old is brought to see the GP with multiple pearly raised papules with central umbilications. They have been there for more than a month on his torso and upper legs. His mother is worried he has warts. What is the most likely diagnosis?
On a newborn baby check of an Asian, 36-hour-old baby you note a large bruise coloured area on the buttocks and lower back which seems non-tender. The mother does not know how it got there. He is handling well and the rest of the baby check is unremarkable. What is the most likely explanation?
A 5-day-old baby is brought to see the GP because she has had a rash for the past 3 days which started on her chest, is spreading to her face and getting worse. On examination she handles well and is alert. There is an erythematous rash on her face, torso and right arm with little pustules. What is the most likely diagnosis?
A 10-year-old girl was diagnosed with diabetes 1 year ago. She has been compliant with her insulin regimen and her HBA1c is 6 per cent. She is attending her annual diabetic review and has been asking about why she has diabetes. What is the aetiology of type 1 diabetes?
A 2-year-old girl was brought by her mother to accident and emergency after 4 days of vomiting and abdominal pain. She had brought her in 2 days ago after developing a cold and was discharged home and diagnosed with a ‘tummy bug’. On examination, she was drowsy, had dry mucous membranes, deep heavy breathing, cool peripheries and tachycardia. Her mother reports a 1-month history of weight loss, excessive drinking and passing large volumes of urine prior to this episode. Her urine dipstick has ketones and glucose. Her blood gas shows the following: pH 7.10, PCO2 kPa 2.1, PO2 kPa 10.0, BE −12, HCO3– mmol/L 18. What is the most likely diagnosis?
A 3-week-old baby is brought to the ‘prolonged jaundice clinic’. His mother reports he has poor feeding, is not gaining weight appropriately and is more sleepy compared to her previous child. He opens his bowel once a day and is being mix breast and bottle fed. He is floppy, jaundiced, has a large, protruding tongue and a hoarse cry. He had a newborn blood spot screening test done at birth which was normal and he has no dysmorphic features. What is the most likely diagnosis of this child?
A 10-year-old girl with Graves’ disease attends her GP with worsening of her symptoms. She was well controlled on carbimazole and has had relatively few symptoms for the past 6 months. She now has sweats, weight loss, diarrhoea and tremors which are affecting her school performance. What is the next management step?
A 4-year-old boy was diagnosed with nephrotic syndrome 6 months ago and has required a long course of oral corticosteroids to maintain remission of the condition. He has developed truncal obesity and you are concerned he may be developing Cushing’s syndrome. Which of the following is not a complication of Cushing’s syndrome?
A 13-year-old girl has presented to her GP with her mother with concerns that she is the shortest in her class at school. She has always been ‘on the small side’ according to her mother, despite eating well. When you examine her you find she is hypertensive but has no cardiac murmur. Respiratory and abdominal systems are normal. She has no signs of pubertal development and you notice she has widely spaced nipples and a low hair line. You are considering the diagnosis of Turner’s syndrome. What is the most appropriate diagnostic investigation?
You are asked to examine a tall 15-year-old boy. His height is above the 98th centile for his age and he has other concerns about the development of breast tissue. He was told this was normal as he develops through puberty but his father states he has no facial or underarm hair. Jake allows a brief examination of his genitalia and you note he has a small penis and testicular volume. He has no arachnodactyly or visual problems. What is the most likely diagnosis?
A 16-year-old boy attends your GP clinic for the first time with his father. He has recently moved to the area. His father is concerned that he is shorter than his peers at school and he frequently complains about being bullied. On further questioning there is no evidence of chronic illness or familial illness and he eats a balanced diet. His weight is on the 25th centile and his height is on the 10th centile. On examination he has no evidence of facial, axillary or pubic hair, his testes are both descended and are <4 mL volume. What is the most likely cause of his delayed puberty?
A 6-year-old girl has presented to her GP with a rapid increase in growth. Her mother is also concerned that she seems to have developed pubic and axillary hair and breast development prior to this but thought it would go away. She has no history of trauma and has reported problems with her vision. Her levels of gonadotrophin-releasing hormone (GnRH), follicle-stimulating hormone (FSH), luteinizing hormone (LH) and oestrogen are high. You are concerned that she may have a pituitary tumour. What is the likely visual field defect?
A 9-year-old girl presents to accident and emergency with fever, vomiting and dysuria. She is wearing a steroid bracelet and has a steroid card stating she is on daily prednisolone for severe asthma and eczema and is therefore at risk of adrenal suppression. She is tachycardic at 140 bpm and you are concerned that her blood pressure is low. Her capillary glucose is 3.0 mmol/L. What is the single most important investigation?
A 3 year old is brought to accident and emergency by his parents because he has not been walking for the past day and refuses to stand. He is normally fit and healthy but he did have antibiotics for tonsillitis 2 weeks ago. They do not think he has had any injuries but he attends daycare and something could have happened there. He is up to date with his immunizations and his parents have no concerns with his development. On examination he looks well, is apyrexial, with a heart rate of 120 and respiratory rate of 26 with no bruising. His knees are normal on examination and the hips have a full range of movement except he cries on external rotation of the right hip. There are no deformities seen on x-ray of the hips and knees. After some paracetamol he manages to stand and take a few antalgic steps with encouragement, limping on the right leg. What is the most likely diagnosis?
A 6-year-old girl is taken to see her GP because she is complaining of knee and elbow pains frequently. Her mother thinks it is worst after her ballet classes and when she gets home from school. She denies stiffness or pain in the mornings. Her mother has been administrating paracetamol several times a week and is worried that this is too much to be giving a child. On examination, the child looks well and has full range of movement of her joints with evidence of hyperextension. There are no swollen joints or effusions present and she is non-tender on examination. What is the most likely diagnosis?
A 14-year-old slightly overweight boy is brought into accident and emergency from a football match where he slipped and fell but was unable to get back up due to pain in his right leg, which is now looking shortened and externally rotated. X-rays show the right femur to be disconnected from the femoral head almost completely at the level of the epiphysis. What is the most appropriate management?
What is a greenstick fracture?
A 2-year-old boy is brought to accident and emergency for the sixth time and is found to have a right-sided non-displaced transverse fracture of his tibia. His parents state that he was running in the living room and tripped landing on a toy truck. He has broken his other leg twice, several fingers and his right arm previously. He appears healthy, is well dressed and his growth is normal. His mother is very upset, she is 5 months pregnant with their second child and her anomaly scan yesterday suggested the baby has a broken leg. What is the most likely explanation for these fractures?
A 2-month-old baby is brought in by the babysitter because he has been crying since she arrived to look after him and his right leg looks swollen. He is the only child living in the household. She does not think he is moving it and is worried it is injured. On examination he is miserable, his heart rate is 160, respiratory rate of 56, and capillary refill is less than 2 seconds. He has a swollen right thigh. He cries more when that leg is examined. You note a yellow bruise on his left thigh and two purple bruises on either arm. X-rays show a fracture of the right femur but the arms appear intact. A chest x-ray shows three healing posterior rib fractures. You are highly suspicious of non-accidental injury. What is the most appropriate management?
On a newborn baby screening examination, you see a baby girl born by elective caesarean section for breech presentation. This is her mother’s first child. The examination is normal except for a clunk felt on Barlow’s test and a relocation click on Ortolani’s manoeuvre on the right side. What is the next step in management?
Which of the following is not a correct match?
A 4 year old is brought to accident and emergency acutely unwell and refusing to walk for the past 2 days. Her parents are not aware of any recent injuries. On examination, she is pyrexial (T = 39.2°C), capillary refill 3 seconds centrally, heart rate 150 beats per minute, respiratory rate 40 breaths per minute. Her right thigh is swollen and slightly erythematous but too tender to examine fully. An x-ray of the hip and femur shows soft tissue swelling surrounding the proximal femur but the bones look normal. An urgent MRI shows a periosteal reaction in the proximal femur with extensive inflammation in the surrounding soft tissues. What is the most likely diagnosis?
A 5-year-old is referred to paediatrics due to concerns initially raised by his school teacher that he is weak and clumsy. On examination he has wasting of his quadriceps and walks in a waddling gait. His blood creatine kinase is 1600 mmol/L (normal is 24–190). What is the most likely diagnosis?
A 3 year old is brought into accident and emergency by ambulance following a generalized tonic clonic seizure that lasted 2 minutes. She did not require any treatment to stop the seizure but on arrival the ambulance crew measured her temperature as 39.2°C and gave paracetamol. She is now apyrexial with a heart rate of 140, respiratory rate of 30 and capillary refill less than 2 seconds. On examination she has red enlarged tonsils with no pus, no neck stiffness or rash. What is the most appropriate management?
A 7 year old is referred to neurology due to frequent episodes of day-dreaming at school where she is unresponsive. She is falling behind in her work because of this. An electroencephalograph (EEG) shows three spike waves per second activity in all leads. What is the most likely diagnosis?
A mother brings her 2-year-old daughter to the GP on a Monday morning. Over the weekend she became very upset on being told ‘no’. She was screaming and then held her breath, went blue and fainted. She woke up quickly and seemed okay afterwards. However, it has just happened again this morning when she found some scissors and her mother took them away. On this occasion she had a brief generalized convulsion lasting about 10 seconds. What is the most likely explanation?
A mother with known placenta praevia with heavy vaginal bleeding was rushed into the labour ward and delivered by emergency caesarean section at 35 weeks’ gestation. Pre-delivery the fetus was bradycardic and after birth APGARs were three at 1 minute, five at 5 minutes and nine at 10 minutes. Thirty-six hours later on the special care baby unit the baby is irritable and requiring nasogastric tube feeds as he is not sucking well. The tone in his upper limbs is reduced and an EEG showed seizure activity which has been controlled by intravenous phenobarbitone. His cranial ultrasound is normal. His blood sugar monitoring is between 3.5 and 5 mmol/L, C-reactive protein (CRP) was less than 5 mg/L and is 7 mg/L today. He is apyrexial. What is the most likely diagnosis?
A 15-year-old girl comes to accident and emergency complaining of sudden right arm weakness and double vision. Last week she was incontinent of urine twice. She is normally fit and well. On examination she has a left-sided 6th nerve palsy and four out of five power in her right arm. The examination is otherwise unremarkable. An MRI head shows multiple hyperintense, inflammatory, white matter lesions. What is the most likely diagnosis?
A 15-year-old girl is brought into accident and emergency from school having disclosed to a friend that she took 10 paracetamol tablets last night. Her blood level of paracetamol is below the treatment line, her liver function tests and clotting are normal. Her father died of a brain tumour 3 years ago and her mother is being treated for reactive depression. The girl tells you that she has been feeling low lately, particularly because she does not think she will do well in her up-coming exams. She regrets taking the tablets and does not think she will do it again. What is the most appropriate management?
Which of the following is not a feature of a UMN lesion?
Which for the following is not a feature of raised ICP?
A 10-year-old girl with sickle cell disease presents to her GP on Monday morning complaining of weakness in her right leg. She says she collapsed on Saturday afternoon and has not felt right since. What is the most likely diagnosis?
A 6-year-old boy is registering with a new GP, having just moved to the area. He is in a wheelchair but is able to mobilize with a fast scissoring gait over short distances. He has increased tone in his legs and has scars from previous tendon release surgeries. His upper limbs are normal. His mother says that his school performance is good and he is writing well. She thinks he was going to have a Statement of Special Educational Needs assessment before they moved. As the GP, what is the most appropriate next step in management?
A 6-year-old boy is taken to see the GP by his mother because he has been getting severe abdominal pains, sometimes with vomiting and yesterday with a headache as well. He has no diarrhoea or constipation. His growth and examination are normal. He has no significant past medical history. In his family history, his maternal grandfather recently died of gastric cancer and mum’s migraines have been worse since his death. She is worried her son is getting gastric cancer too. What is the most likely diagnosis?
Which of the following is not the correct side effect of anti-epileptic medicine?
A 13-year-old Somali girl presents to accident and emergency with a 1-month history of headaches, weight loss and night sweats. Her father is concerned that she seems confused and is more unwell with her headache despite paracetamol. She was born in the UK and has had all her immunizations. She travelled to Somalia 6 months ago. The rest of the family is well although dad has a cough. On examination she is thin and looks unwell but is neurologically intact with no abnormal findings on clinical examination. Which diagnosis needs to be ruled out first?
A 4-month-old baby being investigated for infantile spasms is noted to have an ash leaf macule on his back under Wood’s light. His EEG shows hypsarrythmia. The report of his MRI brain states there are subependymal nodules. What is the diagnosis?
A 3 month old is brought into accident and emergency with a generalized tonic clonic seizure. She is apyrexial and the seizure stopped after 15 minutes with rectal diazepam given by the ambulance crew. Her heart rate is 130, respiratory rate of 36 and capillary refill is less than 2 seconds. On examination she is drowsy, has a port wine stain on her forehead but is otherwise normal on examination. What is the most likely cause of her seizure?
A 79-year-old woman with a diagnosis of Alzheimer’s disease is causing concern as she is constantly getting lost on the way back from the local shop to her home, which is only a short walk and one that she has done nearly every day for 20 years. What sort of memory disturbance does this represent?
A 72-year-old woman who suffers from Alzheimer’s disease is asked who the Prime Minister was during the Second World War, to which she replies ‘Winston Churchill’. She is then asked where she lived during the war, to which she answers ‘Winston Churchill’. What phenomenon is being described here?
A young woman wakes from a nightmare and sees her dressing gown hanging from the door, which she mistakes as an assailant. What is being described here?
A young man with schizophrenia describes how he can hear the secret service in their base in Finland discussing their plans to assassinate him. What is this phenomenon known as?
A 28-year-old man is diagnosed with schizophrenia, with the belief that he has been targeted for extermination by a religious cult who have implanted tiny electrical ‘ants’ into his fingernails. When asked when he knew this, he said he had seen a magazine story 3 months ago on ‘retiring to the country’ and immediately felt this was a covert message from the cult that he should be ‘retired’. There was no evidence of delusions prior to this. What is being described here?
A 48-year-old man with poorly controlled schizophrenia is admitted to the ward. He appears confused and he is difficult to interview. On asking him why he is in hospital, he replies, ‘Jealousy, the Collaborative, collaborate and dissipate. What’s in my fridge? It isn’t my time’. How would you describe this type of thinking?
Which of the following is not a first-rank symptom of schizophrenia as described by Schneider?
A 72-year-old man with Parkinson’s dementia is seen in clinic. He is asked how he is feeling, to which he replies, ‘I feel fantastic...tic...tic...tic...tic...’. What is the name for this type of speech abnormality?
A 26-year-old man is seen by his GP. For the last few months, he has become increasingly concerned about a mole on his cheek, which he feels has got bigger and people are noticing it more. Over the last week he has become convinced that people are laughing at it when he passes them. He has a thought in his head of ‘you’re so ugly, look at the size of that mole’. The patient does not feel he knows where the thought comes from, but it does not seem to be his. He wonders if someone has planted the thought there. The GP does not feel the mole is in any way abnormally sized or has other unusual features. What is the most likely aetiology of these symptoms?
Which of the following is not a core symptom of depression as defined by ICD-10?
A 42-year-old man sees his GP after witnessing a horrific motorway pile-up. For the last 6 weeks he has been experiencing recurrent and intrusive images of the event where he relives what happened, both at night and during the day. At night he is also having vivid nightmares about the crash which is now stopping him from going to sleep. He has not driven his car since, although he himself was not involved in the crash. Every time a car starts he jumps and becomes extremely upset. His mood is low and he feels disconnected from his wife and children and he has been thinking about killing himself. What symptom is not being described here?
What is the most likely diagnosis in the case described in the previous question?
A 49-year-old woman with schizophrenia is admitted to the psychiatric unit in a mute state. She is staring blankly ahead and not responding to any commands. She is not eating or drinking and looks dehydrated. Which of the following is the least likely to be observed in catatonia?
Which of the following statements regarding the two classification systems in psychiatry (ICD-10 and DSM-IV) is false? Note this refers specifically to the section in ICD-10 related to psychiatry and mental health.
Which of the following would be the best definition of the term ‘loosening of associations’?
A man is admitted to accident and emergency after being found semi-conscious in the street. He is unkempt and does not have any information on his person; he appears to be street homeless. In accident and emergency he has a tonic clonic seizure which is self-limiting after 3 minutes. The man is post-ictal for a short time but soon becomes restless, tremulous and sweaty. His speech is rambling, and he complains about the bed sheets being filthy and ‘filled with mites’. He is tachycardic with a blood pressure of 186/114 mmHg. What is the most likely diagnosis?
You order a full set of bloods on this man. Which of the following results would be most indicative of the underlying cause of his delirium?
A 73-year-old woman is admitted to hospital with an infective exacerbation of chronic obstructive pulmonary disease (COPD). Apart from COPD and hypertension she has no other medical problems. On the third day of her admission, she becomes acutely confused. During the night she is awake, shouting constantly for her husband, claiming that the nurses are prison guards and that they are keeping her against her will. She is slightly calmer the day after. You are the FY1 on call and are asked to come and see her over the weekend as the nurses are worried it will happen again at night. What should your initial management be?
Which of the following medications is most likely to be associated with an organic depressive disorder?
A 27-year-old man is involved in a road traffic accident. During rehabilitation his family have become very upset as they feel he has ‘changed’. They report that his concentration is poor and at times he is saying very hurtful things to his wife, which they say is extremely out of character. He has also begun eating large quantities of junk food, whereas before he was extremely fit and careful with his diet. Which part of the brain is most likely to have suffered an injury?
A 28-year-old woman is admitted to hospital systemically very unwell, with a reduced level of consciousness, headache, fever, nausea and vomiting and dysphasia. This is followed by several seizures. Initial cerebrospinal fluid (CSF) analysis shows the CSF is clear, with raised protein, raised mononuclear cell count, no polymorphs and normal glucose. Her partner says that for the preceding few days she had been acting strangely, seeing things that were not there, accusing him of leaving the gas on and getting very agitated. She then became drowsy and he called the ambulance. Your initial management should be based on which being the most likely diagnosis?
A 76-year-old man with squamous cell lung carcinoma attends accident and emergency with his wife who is his full-time carer. She has become concerned as he has become extremely depressed over the last couple of weeks, along with being extremely thirsty and having little energy. Up until then he was coping very well with his diagnosis. What is the most likely cause of these symptoms?
A 14-year-old boy, with no prior psychiatric or medical history, is noted to be seriously slipping in his A-level course work, after previously being a ‘Grade A’ student. He has also started behaving recklessly, going out late whereas previously he had been shy with few friends. He is getting into frequent fights at school. Other changes include the onset of tremor and strange writhing movements in his arms. His mother has also noticed that his skin appears to have taken on a yellow tinge. What is the most likely diagnosis?
Which of the following is the most common psychiatric manifestation following stroke?
A 38-year-old man is admitted with a several week history of rapidly deteriorating memory, which he covered to some extent with extensive confabulation. He was also found to be sleeping, drinking and eating excessively. On examination he was pyrexial. His blood work showed a markedly raised serum osmolality. An MRI shows an intracranial mass. Where is the most likely anatomical location for this lesion?
A 34-year-old woman presents to accident and emergency claiming that the devil has returned to earth and is hunting her through her neighbours, who are recording her every movement. The psychiatric assessment shows florid delusions and auditory hallucinations. She has no past psychiatric history. Her husband tells you that she was fine up until 2 weeks ago. Her hands have also been shaking and she has complained that the devil has been torturing her muscles. She has widespread lymphadenopathy and an enlarged spleen. An unusual rash is present across her cheeks and nose, which she says is the brand of the devil. What is the most likely diagnosis?
Which of the following vitamin deficiencies is most likely to lead to a triad of gastrointestinal disturbance, dermatological symptoms and a heterogeneous constellation of psychiatric symptoms?
Which of the following statements regarding neuropsychiatric manifestations of epilepsy is correct?
Which of the following regarding early-onset dementia (or young-onset dementia (YOD)) is correct?
A 19-year-old white woman presents to accident and emergency with abdominal pain, arm weakness and diminished reflexes. She is also extremely agitated and is responding to auditory hallucinations. You are unable to get a history from her, and you call her GP – there is little of note in her history, although she has only been in the practice for a few months as she is a first year student. The only recent entry is a new prescription for the oral contraceptive pill (OCP). What is the most likely diagnosis?
A 24-year-old student presents with a 3-month history of social withdrawal and low mood. She is difficult to interview because she talks about random themes and has difficulty answering questions. She has vague paranoid ideation. She is childish and pulls faces at you during the interview. The most likely diagnosis is:
What is the lifetime prevalence of schizophrenia in the UK?
A 19-year-old identical twin is diagnosed with schizophrenia. His mother makes an appointment to see you at the GP practice and asks what the likelihood is of his twin developing schizophrenia. What should you tell her?
A 19-year-old man with schizophrenia is brought to accident and emergency by his sister as he has become unwell over the last few days. He has recently been started on risperidone. He is confused, sweaty and tremulous. On examination the signs include tachycardia, low blood pressure, pyrexia and lead-pipe rigidity. His Glasgow Coma Scale score is decreased at 12/15. What is the most likely diagnosis?
A 23-year-old man is diagnosed with schizophrenia. He has had florid persecutory beliefs and auditory hallucinations for the past 3 months. In terms of medical history he has poorly controlled insulin-dependent diabetes and is obese. On admission to hospital he was so distressed he required intramuscular rapid tranquilization. On administration of 5 mg of haloperidol, he developed an acute dystonia in his neck muscles which was excruciatingly painful. What would be the most appropriate drug to commence to control his schizophrenia?
A 24-year-old man with a diagnosis of schizophrenia, last admitted 6 months ago under Section, is brought in by police to the Mental Health Unit under Section 136. He has been harassing his ex-girlfriend with constant threatening phone calls and turning up at her house. He says he believes she is twisting his bones at night, preventing him sleeping and causing him massive pain, through witchcraft. He states that he is going to kill her if it goes on one more night and has purchased a special knife from a ‘witchcraft’ shop on the internet. He is experiencing auditory hallucinations directing him in the best way to use the knife against her. Against the advice of his consultant he has recently stopped his medication, which usually keeps him well. His symptoms typically follow these themes of violence and the supernatural when unwell. He claims that being in hospital will just allow her to target him more easily and will not stay voluntarily. What Section of the Mental Health Act (MHA) is most likely to be appropriate in this case?
The man described above is admitted under Section 3 of the Mental Health Act. On admission to the ward, he is acutely disturbed and becomes violent towards others and himself. He has slapped a member of staff. Staff try to calm him down but it is felt that the risks are escalating. He was prescribed 2 mg lorazepam orally which he has spat into the nurse’s face. He has no prior recorded adverse drug reactions. What is the most appropriate pharmacological management of the patient?
A 22-year-old man with paranoid schizophrenia has been treated with three different antipsychotics and remains unwell. His team decide to prescribe clozapine which he has now been on for 3 weeks. He comes in for his regular blood test and the nurse in the clozapine clinic asks the junior doctor to see him as he appears unwell. On examination, he is sweaty and tachycardic with a temperature of 38.5°C. He has no chest pain but is coughing purulent sputum. What would the most likely isolated abnormality be on blood testing?
A 54-year-old man with schizophrenia has been on depot antipsychotics for the last 27 years as he hates taking tablets and has stopped them in the past. He has not been unwell in terms of his schizophrenia for the last decade. His community psychiatric nurse notices that he has developed odd movements around his mouth over the last few months, where he purses and smacks his lips. It is causing him difficulty speaking and it is distressing for him and his family. Which is the most appropriate course of action for managing this symptom?
A 22-year-old single man is diagnosed with schizophrenia. This is followed by a very rapid psychotic breakdown characterized by well-defined persecutory delusions. There is no mood component to his symptoms. He has shown a poor response to treatment. Which of the following indicates a positive prognostic feature of this man’s illness?
A 38-year-old single woman is arrested outside the house of a celebrity TV chef after shouting outside all night. On interview she claims that the man has declared his love for her several times but is being prevented from seeing her by his wife who is keeping him handcuffed inside. She states it is he that has made several advances to her by sending her special messages when he is cooking on television. What syndrome or symptom is being described here?
A 27-year-old man has been started on haloperidol, a ‘first-generation’ antipsychotic, for control of his symptoms of schizophrenia. A few weeks later he comes to his GP in a highly embarrassed state, claiming that the CIA are experimenting on him, turning him into a woman. When the GP asks how he knows this, the man states that he has noticed his chest growing into ‘breasts’ and he can no longer get an erection with his girlfriend. What is the most likely cause of these symptoms?
Which of the following is not recognized as a diagnostic feature of schizophrenia according to ICD-10?
A 28-year-old woman presents in the GP surgery. She is over-talkative and over-familiar with you. It is difficult to get a full history, but it seems for the last 4 weeks she has been elated and experiencing voices telling her that her mother was a descendant of the Virgin Mary and that she is a female ‘second coming’. This was the result of an experiment by the Nazi party who genetically engineered her grandparents. She believes that remnants of the Nazi party are now controlling her arms and legs, which results in her alternately trying to hug you and then kicking out at the desk. What is the most likely diagnosis?
Which of the following is the least likely to be a side effect of antipsychotic treatment?
A 35-year-old woman complains of low mood after the death of her husband in a car accident. She does not speak and remains immobile for long periods. What is the most likely diagnosis?
A 45-year-old female has had persistent mild depressive features since her late teens. She sometimes experiences loss of energy and tearfulness. She believes her low mood began after she was abused by her step-father as a teenager. She has no other symptoms. What is the most likely diagnosis?
What is the most likely condition causing a depressive episode in a 76-year-old man with a history of smoking and hypertension?
A 35-year-old female visits her GP complaining of low mood and weight loss. On questioning she also experiences fatigue which is exacerbated by pain in her legs. Blood tests reveal high potassium. Which of the following is most likely to cause her depression?
A 43-year-old female visits her GP complaining of a 4-week history of fever, fatigue, low mood and lower back pain. She had visited China in the previous month and mentioned she was drinking plenty of goat’s milk as this was the only type of milk available. What is the most likely infective cause?
A 35-year-old woman has had a low mood for 2 months associated with fever, fatigue and joint pain. She has a rash on her face which gets worse with exposure to the sun. What is the most likely cause of her low mood?
Which of the following is most likely to cause depression?
A 25-year-old woman visits her GP complaining of low mood, fatigue and weight gain. She is observed to be wearing several layers despite its being a warm day. On examination she is found to have a pulse rate of 55. What is the most likely underlying diagnosis?
A 45-year-old woman is taken to see a GP by her husband. He mentions that his wife has been irritable for the past 2 weeks. She is not sleeping well and on examination she has pressure of speech and mild elation. She is not hallucinating and there is no evidence of delusional thinking. What is the most likely diagnosis?
A 25-year-old man with bipolar disorder is seen on the psychiatric ward by a medical student. Taking a history is not easy, as the patient continually wants to speak. The patient is difficult to interrupt. What feature of mania is demonstrated by this patient?
A 25-year-old man is seen in accident and emergency with an elevated mood and mild signs of meningism. He has a low grade fever and a bulls eye rash is seen on his left lower arm. What is the most likely diagnosis?
According to the International Classification of Diseases, which of the following is a core feature of depression?
Which of the following antidepressants is not a serotonin specific reuptake inhibitor (SSRI)?
Which of the following is a significant risk factor for depression?
Which of the following statements is most consistent with a diagnosis of psychotic depression?
Which of the following is the most significant risk factor for completed suicide?
Which of the following symptoms is a recognized psychological symptom of depression?
Which of the following is not a typical feature of anxiety?
A 46-year-old woman is referred to secondary psychiatric services by her GP. Over the last 6 months she has suffered multiple losses, including the death of her sister and a close friend. She lives alone with few social contacts. She has become extremely withdrawn, and is leaving the house less, stating that she gets ‘terrified that I won’t be able to get back to my house’. She reports that when she does go out, she feels breathless, sweaty and like ‘she might faint and make a fool of myself’. What is the most likely diagnosis?
A woman is diagnosed with agoraphobia. She is willing to try any form of treatment as her condition is very disabling. Which of the following management options would not be considered appropriate in the overall management of agoraphobia first line?
Which of the following statements concerning social phobia is correct?
Which of the following statements regarding generalized anxiety disorder (GAD) is incorrect?
Which of the following statements regarding theories of anxiety is correct?
Which of the following statements regarding OCD is correct?
Which one of the following statements regarding OCD is incorrect?
Which of the following statements regarding somatoform and dissociative disorders is correct?
A 42-year-old man is involved in a serious road traffic accident caused by a drunk driver. He is hospitalized for several weeks. Following discharge, friends and family notice that he is not going out, has become withdrawn and appears frightened and anxious all the time. He reluctantly agrees to see his GP. Which of the following would not be consistent with a diagnosis of post-traumatic stress disorder (PTSD)?
Which of the following findings on a MRI scan would be most consistent with a diagnosis of early Alzheimer’s disease?
Which of the following modes of action of currently available pharmacological agents are thought to target some of the symptoms of dementia?
A 79-year-old married woman comes to see her GP with her husband. The husband reports his wife has a history of several months of deteriorating memory and is now forgetting names and faces. He also explains that at times she seems much more lucid, but there are occasions when she becomes very forgetful and confused, sometimes saying there are people sat in the living room with them, which the patient and her husband find distressing. More recently she has developed a tremor in her left hand. What is the most likely diagnosis?
Which of the following features would suggest a diagnosis of depression rather than dementia in a patient presenting with memory loss?
Which of the following statements most accurately reflects depression in older age?
A 90-year-old woman is admitted to the psychiatric inpatient unit with severe depression. She has the following medical history: end-stage chronic renal failure, hypertension, type 2 diabetes controlled with oral hypoglycaemics, and has had a stroke 3 years ago leaving her with some slight speech slurring. Which of the following statements is false?
An 84-year-old man is brought in to the psychiatric unit with a diagnosis of severe depression with psychotic symptoms. He has had three previous admissions with very similar symptoms. During his admission he begins voicing his desire to leave the ward, claiming that the devil is possessing all of the staff and patients, and that he is next. He claims that if he isn’t allowed to leave, he will do whatever he can to escape the devil, even if this means ending his life. He has had to be moved away from the door after trying to follow visitors out of the ward. What would be the most appropriate course of action?
Which of the following statements most accurately reflects manic syndromes in older age?
An 80-year-old woman with a past medical history including hypertension, diabetes and macular degeneration is admitted to accident and emergency complaining of frightening images of birds swooping around her flat day and night. At first she thought they were real but now realizes they could not be. She has no past psychiatric history and apart from being very tearful about the images, there is nothing else of note in the mental state. What is the most likely diagnosis?
A 74-year-old widowed woman, previously fit and well with no past psychiatric history, presents to her GP to ‘have it out once and for all about these bloody neighbours’. She says for the last month her neighbours have been spying on her and are leaking radiation through her ceiling which is making her cough incessantly. On examination she does indeed have a severe cough and has lost weight since her last appointment 3 months ago. Otherwise she looks fairly healthy and well kempt, with an Mini-Mental State Examination (MMSE) of 29/30. Which of the following statements is the most accurate?
Which of the following statements concerning alcohol misuse in older age is the most accurate?
Which of the following statements concerning anxiety in older age is the most accurate?
A 71-year-old man, previously fit and well, presents to his GP with his wife who states he has ‘lost his marbles’ over the last 2 months, with worsening memory loss. He scores 21/30 on the MMSE, losing points mainly on recall as well as dysphasia. His wife has also noticed that he has lost weight. Routine blood tests show the following:
Na+: 129 mmol/L
K+: 4.4 mmol/L
Adjusted Ca2+: 3.1 mmol/L
What is the most likely diagnosis?
A 90-year-old woman is in hospital with late-stage colon cancer which has metastasized. She has been remarkably well all her life before being diagnosed with cancer. She lost her husband 3 years ago but has a supportive family. On the ward, she develops a chest infection. The consultant wants to start her on antibiotics, but she says she does not want them. The consultant asks for a psychiatric opinion, worried that she is depressed. However, the psychiatrist reports she is not depressed and is fully competent to make this decision and is choosing how to die as she wishes. What ethical concept is best described here?
Which of the following statements is true about medicines use in older age?
In the general diagnosis of personality disorder according to ICD-10, which of the following is not necessary for a diagnosis?
Regarding personality and its development, which of the following statements is false?
Which of the following is least likely to predict dangerous behaviour?
A 22-year-old woman with a diagnosis of borderline personality disorder attends accident and emergency after saying she has taken an overdose of paracetamol following an argument with her mother. She is an outpatient at the local personality disorder service where she has a key worker. This is her fourth attendance in accident and emergency for similar reasons in the last 6 weeks. A full assessment reveals no evidence of depression. Her blood results reveal low levels of paracetamol. She does not want to die but cannot say she will not try and harm herself again. What would the most appropriate management be?
A 29-year-old man is arrested for aggravated assault on a former girlfriend. It is his ninth offence of a similar nature. The court asks for a psychiatric opinion. He is noted to be emotionally cold with an extremely reduced tolerance to frustration. He feels no remorse for his actions, blaming his girlfriend for ‘putting it about’. What is the most likely diagnosis?
A 68-year-old woman attends her GP following the death of her husband. She is tearful but ‘doesn’t want to bother the doctor’. The GP notices that she says yes to every suggestion and she says she does not know how to cope as her husband did everything for her except the cooking. The GP feels very helpless and somewhat irritated by the end of the conversation. What is the most likely diagnosis?
A 19-year-old man is referred to the local psychiatric community team as his new GP is worried he is schizophrenic. The letter states that he is ‘extremely odd, and does not seem to have an emotional response to anything’. On assessment, he states he has only come to understand the ‘psychiatric care pathway’ a little more but does not feel he has any problems. He seems aloof and disdainful of the psychiatrist. He appears to have few hobbies except for inventing his own mathematical equations. What is the most likely diagnosis?
A 52-year-old woman comes in to her GP with a swollen knee which appears to be osteoarthritic. During the assessment, however, she quizzes the GP on every little detail of what he is doing. She began the interview by saying how disappointed she was that the GP was running twelve and a half minutes late and that her schedule had been ruined as a result. She asked several times about minute details concerning the referral process. What is the most likely diagnosis?
A 23-year-old woman is referred to the pastoral services at her college because of concerns over her behaviour. She is reported to have episodic outbursts of rage towards her classmates, although she acts in a flirtatious and fawning way towards her male tutors. She has been admitted twice with impulsive self-harming attempts. She has become obsessed with one of the more popular girls in the class, adopting a similar dress sense and texting her often. When she was told by the girl to leave her alone, she became enraged. What is the most likely diagnosis?
Which of the following statements regarding management of personality disorder is correct?
In the structural model proposed by Freud, what is the term used to define the mainly conscious part of the mind that negotiates between the inner wishes and needs and the external world?
A 42-year-old woman suffers a painful breakup with her long-term partner after finding him in bed with another man. She finds the situation, including telling her friends and family, extremely difficult. One year later she is asked how she is feeling. She denies any knowledge of seeing her partner being unfaithful and says ‘oh, we just had our differences, you know, there’s no hard feelings’. What is this form of defence mechanism called?
Which of the following represents a ‘mature’ (i.e. ‘healthy’) defence mechanism?
The following statement refers to which type of psychotherapy? A type of talking therapy, usually short-term and practical, that aims to change the way individuals think or behave with regards to themselves and others, by exploring erroneous patterns of thoughts, feelings and behaviours.
The following statement refers to which type of psychotherapy? A model of therapy where the interactions and relationships between people are explored as opposed to the inner world of the individual.
A 32-year-old woman is being seen for CBT to treat a depressive episode. During the initial assessment, she tells the therapy that ‘to tell the truth, I’m just a bad person’. How might this statement be named in the CBT formulation?
John, a 19-year-old male sculpture student comes to his GP complaining of problems with sleeping. Over the last few months he has been increasingly preoccupied with counting, and is now checking the light switches and other electrical items over and over again well into the night. He now feels compelled to turn the light switch on and off seven times before he can go to bed. He has never had problems before and this is causing him and his girlfriend considerable distress. What is the most likely effective treatment?
Which of the following statements regarding CBT is false?
A 25-year-old man recently married having abstained from sex until marriage. He reports becoming very anxious during sexual intercourse and is gripped by a ‘fear of failure’. Consequently he finds himself monitoring his performance and as a result he cannot maintain an erection. What is the most likely diagnosis?
A 49-year-old man has been successfully treated for anxiety and depression. He is struggling to reach an orgasm during sex although his sexual desire is normal. What is the most likely cause of his current problem?
A 35-year-old man is picked up by the transport police after reports that he had been rubbing his erect penis against several female passengers on a train. The female victims were unknown to the offender. What is the most likely diagnosis?
A 62-year-old female with a history of rheumatoid arthritis complains that when she attempts to sleep, she feels an urge to move her legs due to uncomfortable sensations. Movement does ease the distress. What is the most likely diagnosis?
A 23-year-old medical student is nearing her final examination. She feels as though she should study as much as possible and consequently has been revising into the early hours of the morning, drinking up to six cups of coffee per day as well as an energy drink. She has little difficulty getting to sleep but wakes intermittently during the night. She does not wake refreshed and feels tired the next day. There are no other symptoms. What is the most likely diagnosis?
A 20-year-old male patient is taken to see a GP by his father. The patient has had three distinct periods of binge eating coupled with long periods (lasting up to 18 hours) of sleep over the past 3 months. Each attack lasts a few days or so and then spontaneously resolves. What is the most likely diagnosis?
A 45-year-old businessman, who travels regularly as part of his work, visits the corporate physician complaining of difficulty in getting to sleep as well as daytime fatigue and reduced performance during presentations. He also has periodic headaches that are relieved by paracetamol. What is the most likely diagnosis?
A 20-year-old female student visits her GP complaining of suddenly waking with a ‘feeling of falling’. What is the likely diagnosis?
Which of the following features would indicate a diagnosis of anorexia nervosa rather than bulimia nervosa?
Which of the following is not a recognized complication of sustained anorexia nervosa?
A 26-year-old male sees his GP. He recently fell and broke his wrist while drunk and is seeing his GP for a follow-up appointment. He has lost his job as he was found drinking vodka from a water bottle. The patient insists his recent problems are down to ‘bad luck’ and not alcohol. What is the most likely diagnosis?
A 40-year-old regular cocaine user was made redundant as an advertising executive 2 weeks ago. He presents with a 6-day history of low mood, anhedonia, irritability and increased appetite and has been feeling generally fatigued. What is the most likely diagnosis?
A 52-year-old confused man is brought to accident and emergency by ambulance after being found on the ground with a head injury. He is known to have alcohol dependence. On examination the patient is obtunded, ataxic and has bilateral weakness in his lateral recti ocular muscles. What is the likely diagnosis?
A 21-year-old university student is at a union event on a Friday night. He becomes aggressive and gets into a fight with a stranger over a spilt drink. What is the most likely diagnosis?
A 42-year-old man with alcohol dependence has gone to extreme lengths to prove his belief that his wife is having an affair with the gardener. The patient has admitted to placing secret surveillance cameras in the home he shares with his wife. What is the most likely diagnosis?
A 34-year-old man with a long history of alcohol dependence is admitted to a hospital ward. Two days later the patient is found to be quadriplegic and can only communicate ‘yes’ and ‘no’ using eye signals. What is the most likely diagnosis?
A 26-year-old Asian man has been drinking a bottle of whisky a day for 10 years. He has had a cough for 6 weeks, with haemoptysis and night sweats. What is the most likely diagnosis?
A 45-year-old woman who lost her family in a road traffic accident 10 years ago has been dependent on alcohol since. She presents to her GP with difficulty swallowing. She is referred to a gastroenterologist who reports the presence of columnar epithelium in the lower oesophagus. What is the most likely diagnosis?
A 30-year-old city executive presents with alcohol dependence. On further questioning it emerges he has always been very anxious about having to give presentations to his colleagues. When he is required to speak in front of them, his heart races, he begins to sweat profusely and feels an urge to leave the stage. He has been drinking vodka in order to suppress these symptoms. What is the most likely diagnosis?
A 30-year-old banker presents to accident and emergency. He is agitated and continuously scratches his skin, complaining there are ‘insects crawling all over him’. His blood pressure is raised and an electrocardiogram reveals a tachyarrhythmia. What is the most likely cause of his symptoms?
A 48-year-old man is seen by his community psychiatric nurse. On questioning he shows evidence of persecutory delusions despite treatment with risperidone. The patient has a history of long-term drug misuse. What is the most likely causative drug?
A 45-year-old man presents to his GP because he has recently become worried about his drinking. He says that drinking red wine can be ‘beneficial’ and drinks four bottles over the course of the week. He is otherwise well but now thinks he should cut down his drinking. What is the next appropriate step in management?
A 40-year-old man is brought into accident and emergency. He is extremely agitated and confused about both time and day and provides a very unfocused history. He is sweating and tachycardic. Later that evening he complains that tiny birds are attacking him. What is the next appropriate step in management?
Which of the following is an atypical (second-generation) antipsychotic drug?
A 29-year-old man is seen by a psychiatrist and commenced on venlafaxine. What class of drug is this?
Which of the following drugs is not used as a mood stabilizer?
Which of the following would be the most appropriate choice in the first line management of new-onset schizophrenia?
Which of the following would not be an appropriate choice in the prophylaxis of bipolar disorder?
Which of the following would be the most appropriate choice of drug for the management of a 55 year old with postoperative delirium who has become extremely agitated?
Which of the following is most likely to cause tardive dyskinesia in a middle-aged man with schziophrenia?
Which of the following primarily acts to increase levels of acetylcholine in the brain?
Which of the following drugs is contraindicated in myasthenia gravis?
A 4-year-old boy is brought into his GP by his parents. They are worried as he is constantly dropping things and trips often, sometimes causing injury. He does not show any affection towards his family and does not play well with others at nursery, although his older sister is a very warm child. He plays with dinosaurs by himself but completely ignores other toys. His speech is relatively normal. What is the most likely diagnosis?
A 12-year-old boy is referred to the child psychiatry service. His behaviour has become so aggressive that he has been excluded from school for assaulting fellow pupils and more recently teachers. He has smashed up several classrooms and the previous week the fire brigade were called as he set fire to his bedroom. He shows no remorse for the way he behaves. What is the most likely diagnosis?
Which of the following would be least appropriate for the first line management of conduct disorder?
Which of the following is not part of the diagnostic criteria for ADHD?
A 9-year-old boy is brought to the GP as he has started wetting the bed, despite being continent for the last 4 years. What is this symptom known as?
A 9-year-old boy is referred to the local child psychiatry service. For the past 18 months he has begun displaying odd speech, with outbursts of strange and sometimes obscene words. More recently he has begun grimacing and blinking excessively. He is unable to control this and it is causing him some distress. What is the most likely diagnosis?
An 11-year-old boy is diagnosed with Gilles de la Tourette syndrome. There is no evidence of any co-morbid diagnosis. What would the most appropriate management be?
Which of the following statements regarding learning disability is correct?
Which of the following is not usually associated with learning disability?
Which of the following statements regarding trisomy 21 is correct?